Sei sulla pagina 1di 98

MUSCULOSKELETAL

I. Definitions....137
II. Evaluation and Examination of the
Musculoskeletal System....141
1. Introduction....141
2. History....141
3. Physical Examination....142
4. Examination of Specific Areas....144
SPORT III. Case Histories....149
A. Shoulder....149
MEDICINE MANUAL 1. Shoulder Dislocation....149
2. Recurrent Subluxation Of The Shoulder....151
3. The Overhead Athletes Shoulder....152
4. Muscle-Tendon Injuries: The Shoulder
Impingement Tendinopathy....156
5. The Acromioclavicular Joint....158
6. Fractures....160
B. Elbow ....161
1. Acute Tennis Elbow Syndrome....161
2. Elbow Dislocation ....164
3. Throwers Elbow....167

4 C. Wrist and Hand....169


1. Distal Radial Fracture....169
2. Sprained Wrist ....171
3. Mallet Finger....173

4 - Injuries to the Musculoskeletal


4. Acute Tenosynovitis of Extensor Tendons ....174
D. Spine ....176
1. Lumbosacral Disc Disease ....176
2. Spondylolysis and Spondylolisthesis ....179

System
3. Facet Joint Syndrome ....182
4. Cervical Nerve Root Injury versus Brachial
Plexus Injury ....184
E. Hip, Pelvis and Thigh ....186
1. Femoral Neck Stress Fracture ....186
2. Apophyseal Injury ....192
3. Hip Pointer ....196
4. Quadriceps Contusion (Myositis Ossificans) ....197
5. Hamstring Strain ....200
6. Femoral Fracture ....201

- 135 -
F. Knee ....204
1. Patellofemoral Pain Syndrome ....204
2. Iliotibial Band Friction Syndrome ....207
3. Acute Non-contact Knee Injury with Haemarthrosis ....209
4. Acute Knee Injury ....211
G. Lower Leg Pain....212
H. Ankle and Foot ....215
1. Ankle ....215
2. Medial Foot Pain ....221
3. Forefoot Pain ....226
4. Rear Foot Pain ....228
I. References....232
4 - Injuries to the Musculoskeletal
System

- 136 -
IOC Sport Medicine Manual 2000
INJURIES TO THE
MUSCULOSKELETAL SYSTEM
I. Definitions
4
Apophysitis - To adapt to bone lengthening, tendon attachment
occurs at the apophysis and its corresponding growth plate. An apophysitis may occur from the traction
of the tendon on the apophysis with excessive muscle contraction during the prepubescent and pubescent
growth periods. Apophysitis may also occur following a direct blow to the apophysis. (A kick to the
posterior calcaneus while playing soccer is a common mechanism). Common sites of apophysitis are
the posterior calcaneus (Severs Disease), tibia tubercle (Osgood-Schlatters Disease), ischial tuberosity
and iliac crest. A possible complication is avulsion of the apophysis.

Apophysitis will present with pain in the area, pain with contact and an increase in pain with use of the
muscle-tendon unit. There will be local tenderness, swelling, and pain with compression of the
apophysis. Radiograph findings may include altered density of the apophysis as well as fragmentation
of the bone tendon junction. Treatment includes relative rest, a strengthening programme, and protection
from direct contact.

Avulsion Fracture - An excessive contraction or stretch of a muscle may cause the bony site of a
tendon attachment to be separated from the rest of the bone. This is called an avulsion fracture and can
be demonstrated by x-ray. In children, many of the muscle attachment sites are associated with a
growth plate (physis). Sudden muscular contraction or stretch may cause the apophysis to separate
from the main bone (eg. the hamstring may pull the ischial tuberosity apophysis from the ischium).
Avulsion fractures can occur both with respect to muscle-tendon unit attachments and ligament
attachments to bone.

Compartment Syndrome - Acute - This may follow serious injury of the leg or arm, and is a
progressive cycle of worsening ischemia. It can also occur as a result of any injury (eg. dislocated
knee) with vascular compromise. Acute compartment syndrome is initiated by bleeding or edema into

4 - Injuries to the Musculoskeletal


the compartment, giving rise to an increase in compartment pressure, which in turn results in reduced
capillary flow and subsequent muscle ischemia. High-risk injuries for this cascade of events include
fractures of the proximal tibia, elbow, and forearm, crushing injuries, or direct blows from contact
sport. Rarely, it can result from excessive unaccustomed exercise, or following contusion in an athlete

System
with anti-inflammatory medication-induced prolonged clotting times.

The athlete can experience paraesthesia within 30 minutes of ischemia, changes in muscle function
within 2 to 4 hours, and myoglobinuria after 4 hours of total ischemia. The classic features of ischemia
include pain, paraesthesia, pallor, paralysis, and no pulse in the extremity. A diagnosis should be made
long before all of these characteristics are present because of the emergency nature of this condition.
A tense, swollen limb, that is unduly painful even without activity, should be evidence for concern.
On examination, exquisite limb tenderness, possible reduction of two-point discrimination on sensory
testing, and pain recreated by passive muscle stretching that is out of proportion to the clinical scenario
warrants re-examination at frequent intervals and pressure measurements. Following ischemia, direct
intracompartmental pressure measurements of higher than 40 mmHg necessitates urgent surgical
decompression of the compartment by open fasciotomy, to avoid potential permanent tissue destruction.

- 137 -
IOC Sport Medicine Manual 2000
Compartment Syndrome - Chronic - This is also known as exercise-induced ischemia, or exertional
compartment syndrome, and results from increased intramuscular pressure during exercise. In the leg,
chronic compartment syndrome often follows a predictable duration of high level repeated dorsi- and
plantar- flexion physical activities, such as walking or running. It is thought that muscle microtrauma,
resulting from such activity, can give rise to an inflammatory reaction of the muscle and capillary bed,
leading to increased blood flow and consequently greater volume and pressure within the compartment.
Pathologically, it presents as thickened fascia. An athlete with this condition will develop pain, swelling,
tenderness, impaired muscle function, and possible numbness and tingling in the distribution of the
peripheral nerve associated with the compartment. Normal resting compartment pressures range from
5-15 mmHg. Following cessation of exercise, pre-exercise resting pressures return within 2-5 minutes.
In individuals presenting with this form of compartment syndrome, resting pressures are occasionally
high (>15 mmHg), and immediate post-exercise pressures are greater than 30 mmHg. There is a
delayed return (> 3 minutes) to resting values.

Dislocation - A dislocation occurs as a result of disruption of the stabilizing structures of a joint. The
articular surfaces are completely displaced and are no longer in contact. Dislocation implies significant
ligamentous disruption (greater than that with subluxation). Many dislocations occur at the time of
the injury but reduce spontaneously and, therefore, the actual dislocation may only be suspected from
the description of the mechanism of injury and the force involved. If ligaments and surrounding soft
tissue supports are damaged, repeated dislocations could occur.

An athlete will present with an acute joint dislocation, often holding the involved limb in a characteristic
position, with joint movement being painful and restricted. The joints outward appearance is abnormal
and bony landmarks may be displaced. One should be suspicious of fracture-dislocation for any
dislocation. If no fracture is involved, the dislocation should be reduced as soon as possible under
controlled conditions, and then immobilized to allow for healing of the soft tissue damage.

In the presence of an acutely dislocated joint, it is imperative to examine the neurovascular status of
the involved limb before and after attempting reduction of the dislocation. It is also helpful to consider
the type, direction, and etiology of the dislocation before relocating the joint.

Hip Flexor Strain, Groin Strain or Lower Abdominal Strain - Another area where there is confusion
4 - Injuries to the Musculoskeletal

about the diagnosis of specific musculotendinous problems is in the groin or lower abdomen. An
athlete may say they have a muscle injury that in fact may refer to many different diagnostic entities
ranging from hip pathology to hernias to osteitis pubis. It is necessary to differentiate between the
multiple different diagnoses that are associated with these commonly used terms.
System

Ligamentous Laxity - Laxity refers to hypermobility of a joint. Laxity can be localized (injury,
infection) or generalized. Generalized joint laxity occurs in approximately 5% of the population and
is inherited. Hypermobile joints are not necessarily unstable to the point where they may sublux or
dislocate.

Ligamentous Instability - This implies an unstable joint, often secondary to disruption of the
ligamentous structures. Joint stability is dependent on the intricate interplay between static (bones,
ligaments, capsules) and dynamic (muscles) structures.

Ligament Sprain - A sprain refers to an injury to a ligament as a result of excessive tension. The
tensile load can result in damage anywhere along the length of the ligament depending on the age of
the athlete. Typically, most ligament sprains occur within the ligament itself rather than at the
attachments in the mature athlete.
- 138 -
IOC Sport Medicine Manual 2000
Ligament sprains are categorized according to the severity of ligament disruption and injury to
associated structures. In addition, the associated presenting symptoms and signs are considered. Based
on these criteria, sprains are classified as mild, moderate, or severe.

Table 4.1 Ligament sprain classification

Mild (first degree) Moderate (second degree) Severe (third degree)

Microscopic ligament Varying degree of tearing in Complete loss of


Pathology stretch, with minimal ligament and capsule. continuity with ligament
or no bleeding. or capsule.
Painful restriction of Localized pain, often with Unstable joint with
movement, with bleeding and bruising, and swelling, no resistance to
Symptoms & Signs
minimal mechanical some joint instability. stress testing.
weakness.
Greater than 6 weeks, may
Healing Time Days to 2 weeks 3 to 6 weeks
not heal on its own.
Rest, followed by guided
range of motion.
Treatment Early range of motion Protected range of motion
May require surgical
repair or reconstruction.

Muscle Contusion - A muscle contusion results from acute direct trauma to the muscle. This
commonly occurs in contact sports following a collision with another player or direct contact with the
ball, puck or stick. There will be tearing to the muscle fibres, haemorrhage and haematoma formation.
Classification of muscle contusions is based on the severity of the trauma and amount of haemorrhage
(mild, moderate and severe) as well as the site of the haematoma (intermuscular and intramuscular).
Intermuscular haematoma occurs near the intermuscular septa and facilitates haematoma resolution.
In contrast, an intramuscular haematoma has increased inflammation and prolonged recovery time

4 - Injuries to the Musculoskeletal


due to slow haematoma dispersion. An acute complication may be acute compartment syndrome.
Chronically there may be persistent pain, re-injury or myositis ossificans.

The patient with a muscle contusion will usually present with pain at the site of injury, swelling,
bruising, and restriction of movement. Tenderness, reduced range of motion and pain with passive

System
stretch will be the relevant physical findings. Initially radiographs will be normal, but in chronic
conditions, calcification may be evident. Treatment will depend on severity, but control of the swelling
and prevention of further haemorrhage will be the first priority. A controlled range of motion and
strengthening programme may be implemented as symptoms subside and exercise tolerance increases.

The most common site of a muscle contusion is the anterior or lateral thigh or quadriceps muscle.

Muscle Strain - A muscle strain injury occurs as a result of tension to the muscle-tendon unit. This
tension can result in simple overstretching of the muscle-tendon unit with pain and no loss of function,
to the extreme of complete muscle or tendon rupture. These injuries typically occur as a result of
eccentric overload where there is tension and lengthening of the muscle at the same time. Major
muscle-tendon ruptures typically occur when there is a rapid change from a concentric contraction
(eg. shortening of the muscle tendon unit) to an eccentric contraction. Under these rapid and excessive

- 139 -
IOC Sport Medicine Manual 2000
loading conditions, the weakest spot in the muscle tendon unit will sustain the most damage. In skeletally
immature individuals, particularly in adolescents, this will occur at the apophysis. In adults, the areas
of weakness are typically the muscle-tendon junction and the tendon itself.

Muscle strains are classified into first, second and third degree injuries. A first-degree strain is a
minor, partial injury to the muscle-tendon unit. There is typically no sign of swelling, but bruising and
tenderness at the area of injury may be evident. The function of the muscle-tendon unit is intact
although painful when resisted contractions are attempted. The second-degree strain is associated
with muscle-tendon fibre damage. This is associated with a greater degree of pain and swelling, bruising
and in some situations, a palpable defect in the muscle-tendon unit. Second-degree strains are associated
with weakness and impaired function. A third degree strain is the complete disruption (a palpable gap
and complete loss of function) of that muscle-tendon unit. If the injury occurs at the muscle-tendon
junction, there typically will be a large amount of bleeding and bruising. If it occurs in the tendon,
there is less associated bruising and bleeding. Although these third degree injuries are associated with
significant pain and sometimes a pop, tear or ripping sensation, they typically are less painful than
second-degree strains after the injury has occurred.

Shoulder Dislocation - A separated shoulder typically refers to a dislocated acromioclavicular joint


in the shoulder, whereas a dislocated shoulder refers to dislocation at the glenohumeral joint. In fact,
both of these injuries could be considered shoulder dislocations in the sense that they both involve
joints that are dislocated and are part of the shoulder girdle itself.

Subluxation - A subluxation is defined as a partial disruption or partial dislocation of the joint, where
the two joint surfaces are still in contact but not in a normal orientation. If it occurs on a recurrent
basis, it can be quite disabling to the athlete, and can lead to dislocation once the ligamentous continuity
is lost.

Tendinopathy - Tendinopathy is a clinical term for all the different anatomic pathologies that may
occur in and/or around the tendon. Tendinopathy will result following acute or chronic overload of the
affected tendon. Examples of tendinopathies include tendinosis, tendinitis, paratendinitis, and
paratendinitis with tendinosis. Common sites of tendinopathies are the patellar tendon, rotator cuff
tendons, posterior tibialis and the Achilles tendon.
4 - Injuries to the Musculoskeletal

The diagnosis of the specific tendinopathies requires direct visualization of the tendon and/or
histological examination. Therefore, a clinical exam may diagnose a tendinopathy, but the precise
pathology may not be confirmed without further evaluation of the injured tendon. There will usually
System

be pain with movement of the joint and weakness with resisted muscular contractions. The tendon
may be tender, swollen, or warm as well as painful. Ultrasonography is the investigation of choice.
Radiographs may reveal calcification of the affected tendon. Treatment is specific to the type of
tendinopathy, but initially will often include non-steroidal anti-inflammatory medications, activity
modification with an incremental stretching and strengthening programme. Complications include a
subsequent partial or complete tear of the tendon.

Tendinosis - It is a degeneration of the intra-tendinous fibres that may result with aging, microtrauma,
and inadequate vascular supply with no signs of an inflammatory response. This is probably the only
clinically relevant chronic tendon lesion.

Tendinitis - It is a degeneration of the tendon with an inflammatory response and may involve a tear
of the tendon (some argue whether this truly exists). Paratendinitis is inflammation of the paratendon
(outer lining of the tendon) only.
- 140 -
IOC Sport Medicine Manual 2000
II. Evaluation and Examination of the Musculoskeletal System

1. Introduction
Prior to commencing a physical examination of an injured athlete, an accurate history is of critical
importance. The history typically establishes the diagnosis and focuses the subsequent physical
examination. The chief complaint is the initial focus of the history, followed by a determination of the
specific mechanism of injury. The outline below demonstrates the appropriate areas that should be
addressed.

In dealing with the injured athlete, there are certain special situations that must be considered. These
would include the acutely traumatized athlete where one should follow a typical advance trauma life
support protocol or at a minimum, follow the ABCs of resuscitation. Another situation would be
where there is a potential cardiorespiratory problem where the ABCs would be instituted, followed
by the possibility of advance cardiac life support protocol. Other special situations would include
assessing the injured athlete on the field of play where there may be specific regulations that place the
sport practitioner in a difficult situation not only because of the potential hostile environment, but
also where decisions and evaluations need to be made on a time-limited basis.

2. History
 Identify the predominant presenting symptom or symptoms (chief complaint).

 Identify the onset of the symptom(s), including the mechanism of injury, whether it is of
acute traumatic onset, overuse or chronic. This initial information will direct the subsequent
flow of the patients interview.

 The severity of the complaint is important to appreciate. A simple way to do this is by asking
the patient to rate their symptom eg. pain, on a scale of 0-10. This can be reassessed on
subsequent visits. It also gives an idea of whether surgery is appropriate.

 The history of present illness should follow. This includes associated symptoms, aggravating
and alleviating factors, the quantity and quality of the symptoms, when and where they occur.

4 - Injuries to the Musculoskeletal


The impact of the problem on activities of daily living, on the patients occupation and lifestyle
including sport and recreation are important to address.

 The past history as it relates to the present injury/illness should be addressed.

System
 The previous treatment and response to treatment should be identified.

 Other joint, bony or regional musculoskeletal involvement needs to be identified.

 Chronological and physiological assessment of age is of both diagnostic and therapeutic


importance.

 Past medical history and family and social history should be identified.

 One should review associated physiological systems, the current medications and the patients
allergies.

- 141 -
IOC Sport Medicine Manual 2000
3. Physical Examination
The environment needs to be comfortable (office set up) in order to carry out an effective examination.
The examination table must allow bilateral examination, as well as the front and back of the patient.

The general physical examination is an important component of the overall physical examination.
This would include vital signs and the assessment of other organ systems as required based upon the
history.

Comparison to the Opposite Side


The best indicator of an abnormality is an objective difference between the affected, abnormal side
compared to the opposite or normal side.

Joint Above and Joint Below


It is necessary to evaluate the surrounding joints when dealing with a specific joint or region. This
will account for referred or related symptoms or problems. The inspection must be active (eg. looking
for things that are relevant to the history and making the diagnosis). The area to be examined must be
exposed in order to see the areas of concern, eg. you must be able to see both the anterior as well as the
posterior aspect of the shoulder girdle to allow for full inspection. Palpate the region to identify areas
of tenderness.

Point of Maximal Tenderness


The point of maximal tenderness associated with knowledge of the anatomy is very likely highly
correlated with the area of pathology (eg. medial joint line tenderness with medial meniscal tear of the
knee).

Simultaneous Examination on Both Sides


By evaluating both sides of the body simultaneously, it is possible to get an immediate comparison.
This procedure also allows the examiner to palpate firmly or resist strength vigorously in order to
detect abnormalities. The patient will also perceive the difference between the two sides in most
cases, rather than what otherwise may be perceived as an over aggressive examiner.

Range of motion must be assessed actively (eg. patient moves themselves) and passively (you move
4 - Injuries to the Musculoskeletal

the joint or limb). Any loss of motion or discrepancies between the two sides should be documented.

Pain Does Not Equal Loss of Range of Motion


If a patient is unable to move a joint due to pain, it does not necessarily mean that the joint does not
System

move.

Normality
In the musculoskeletal system, there is a tremendous range of what is considered normal, especially
with respect to range of motion in a joint, strength, laxity (translation of a joint) and adaptive asymmetry.

Strength Testing
Strength testing relates to a specific movement or function. It is very difficult to isolate one specific
muscle; the examination generally evaluates a group of muscles. The MRC grading system is
universally used with the following designations: 0 = No contraction; 1 = Flicker or trace of contraction;
2 = Active movement with gravity eliminated; 3 = Active movement against gravity; 4 = Active
movement against gravity and resistance (4-, 4, and 4+ are used to indicated slight, moderate, and

- 142 -
IOC Sport Medicine Manual 2000
strong resistance); 5 = Normal power. Quantitative tests of muscle strength may be required in athletes
to identify unilateral weakness that is not evident with manual testing.

Pain Does Not Equal Weakness


When a patient complains of pain during strength testing, any perceived weakness may be an unreliable
sign of true weakness.

Tests
 When testing for stability/instability, you must have a cooperative and relaxed patient.

 When one side of a joint is moved with respect to the other side, the specific magnitude and
direction of the translation is documented. An objective difference between the normal and
abnormal side implies instability.

 If a joint is translated, rotated or placed into a provocative position, the patient may perceive
instability during this manoeuvre. This is the best sign of an unstable joint.

 When performing provocative manoevers such as an apprehension test, you should look at
the patients face, not the joint being tested. You will be able to identify apprehension in the
patients features as an indication when to discontinue the manoeuvre.

 A neurovascular examination should be part of the musculoskeletal evaluation in every patient.

 Each individual joint or region has special tests that are an important part of the examination:
(eg. McMurrays test for meniscal tears in the knee; Thomas test in the hip for flexion
contractures; Thompsons test for Achilles tendon rupture; impingement tests in the shoulder;
and Finkelsteins test for tenosynovitis of the thumb tendons).

 It is necessary to measure leg lengths, girth sizes and range of motion using tape measures,
standing blocks of known height and a goniometer. These measurements must be made with
a specific reference point and the inherent errors should be appreciated.

4 - Injuries to the Musculoskeletal


 To many practitioners, x-ray imaging and soft tissue ultrasonography are an extension of the
physical examination of the musculoskeletal system.

System
Gait Analysis
Examination of the patients walking and running gait is an important aspect of the musculoskeletal
exam. This should include assessment from more than one perspective (eg. from the front and back
and from the side).

Assessment of Function
It is necessary in most circumstances to assess not only specific motions and strength, but also the
actual functions and skills that an individual can demonstrate. This would include assessment of
balance on one leg with the eyes closed, hopping, a single leg squat, jumping, performing a duck walk
or doing a heel raise.

Summary
The outline above describes a generic approach for the examination of the musculoskeletal system.
This can be applied to any specific region of the body. It should be clearly pointed out that as a result

- 143 -
IOC Sport Medicine Manual 2000
of training over a long period of time, the musculoskeletal system adapts to the specific demands of
the sport involved. For example, one should be aware of the normal hypertrophy and changes in the
ranges of motion in the shoulder girdle of a throwing athlete, such as a baseball pitcher. Similarly, one
should be aware of the tremendous flexibility that is typically present in a female gymnast. These
attributes are normal findings and are typical of individuals involved in these sports.

4. Examination of Specific Areas


Spine
The examination of the spine should include the following. The spine should be exposed so that any
deformities can be identified. The patient must be visible from the back. The point of maximum
tenderness should be identified and correlated with the anatomical level. Range of motion should be
assessed in flexion, extension, lateral flexion and rotation in all directions. Combinations of these
motions - for example extension, lateral flexion and rotation to one side - may typically provoke a
symptom such as radicular pain from the cervical spine. A careful neurological examination should be
performed as part of the examination of the spine including gait assessment and coordination. The
neurological findings should be carefully documented.

Spinal problems can be categorized into those involving the musculoskeletal system or the nervous
system or a combination of both. It is important to be able to distinguish between the two because
nervous system problems typically require more in-depth investigation and evaluation.

Shoulder
Examination of the shoulder is typically complex and difficult for most people. Simplifying evaluation
of the shoulder allows for improved documentation, reproducibility and diagnostic accuracy. The
shoulder should be examined from both the front and the back with comparison to the opposite side.
A point of maximum tenderness should be identified in the affected shoulder by concurrent examination
of the opposite shoulder. The range of motion should be documented actively and passively.

At a minimum, the following ranges should be assessed: external rotation with the elbow at the side,
internal rotation with the position of the thumb along the spine, external rotation and internal rotation
in a 90o abducted position, and elevation of the shoulder in the scapular plane.
4 - Injuries to the Musculoskeletal

At a minimum, strength should be assessed in external and internal rotation as well as resisted elevation
in the scapular plane. Strength testing in the functional throwing position should also be considered.
Tests of translation and apprehension should be evaluated in the anterior, posterior and inferior
directions. Tests for impingement should also be assessed but it should be recognized that these are
System

non-specific, particularly in the athletes.

Lower Abdomen and Pelvis including Groin


The lower abdomen, pelvis and groin are closely related from the muscular and skeletal standpoint.
One must consider the intra-abdominal and pelvic organs as potential areas of referred pain. Identifying
the point of maximal tenderness and correlating it with the anatomy is critical in simplifying this
complex region. Assessment of abdominal muscle function through a modified sit up will help identify
those injured structures in the abdominal wall. It is important to assess the range of motion in the hips
as well as function in their adductor and abductor muscles. One should consider both inguinal and
femoral hernias as part of the differential diagnosis and examination. Stressing the bony pelvis in
compression, distraction, and rotation will help sort out problems of the bony pelvis, specifically
those related to the symphysis pubis such as osteitis pubis or stress fractures.

- 144 -
IOC Sport Medicine Manual 2000
Hips
Examination of the hip in the athlete may be difficult due to the typical hypertrophied musculature.
Nevertheless, correlation of the point of maximal tenderness to the functional anatomy is typically
important. This should be followed by functional tests of the injured athlete in a standing position, as
well as assessment of strength. The range of motion examination is critical and any discrepancy from
side to side should be considered abnormal and investigated thoroughly.

Knee
After determining the chief complaint of the athlete, the history of the problem is elicited to determine
if there was a specific injury or a gradual onset of problem as well as the treatments and results to date.
Visual examination of the knee joint with the patient in supine includes identifying effusion, muscular
atrophy, orientation of the patella, bruising, abrasions, and the alignment of the femur and tibia. Begin
the examination around the patella to check laxity of the patella and for patellar and patellar tendon
tenderness. If one demonstrates true apprehension in the patellofemoral joint, this should be considered
abnormal on its own. An effusion may be visible or palpable. In situations where there is a minimal
amount of fluid, one must decrease the volume of the joint by compressing the suprapatellar pouch in
order to identify an effusion. The point of maximal tenderness as always should be correlated with the
anatomy.

The knee should be moved through an active range of flexion and extension and then placed through
full passive range of motion. Two joint muscles are important for knee functional range of motion and
are often strained. Therefore, each of the muscles that cross both the hip and knee or ankle and knee
should be tested independently: hamstrings, rectus femoris, gracillis, tensor fascia lata, and
gastrocnemius muscles.

With the knee flexed at 90, the surface anatomy is evaluated. Specifically, palpate for tenderness over
the menisci and along the course of the ligaments and tendons, as well as on the bones. With the knee
flexed to about 30, the collateral ligaments are stressed by gently rocking the knee into varus and
valgus to test the limits of laxity and compare with the normal contralateral knee. Similarly, at the
same degree of flexion, anterior stress is applied to test the anterior cruciate ligament for the amount
of translation of the tibia upon the femur, as well as the feel of the end point to this stress as to whether

4 - Injuries to the Musculoskeletal


it feels tight or loose (Lachmans test). This test is 90% reliable once some experience is gained.
Anterior stress upon the knee at 90 of flexion (anterior drawer test ) is only 60% reliable for anterior
cruciate ligament tear detection. At 90 of flexion, evaluate any posterior sag of the tibia and then
apply posterior stress to detect any more posterior cruciate ligament laxity (posterior drawer test). If
the patient is comfortable enough, test for rotatory instability by a pivot shift test, a flexion rotation

System
drawer or a jerk test. Subluxation of the tibia anteriorly with relocation to the normal position (the
pivot shift) is an absolute abnormal sign. Isolating the ligament through specific positioning is critical
to identify partial ligament injuries. It should be emphasized that ligament testing is based upon a
careful comparison to an opposite normal side. If the other side is not normal, then it is difficult to
interpret these tests.

The menisci are examined by applying stress tests. One test for meniscus integrity involves moving
the knee into full flexion, then external rotation and extension at the same time as palpating over the
medial meniscus trying to feel a thud or other mechanical event under your finger. If such an event
is present, then there is a very high possibility of an unstable meniscal tear. Similarly, for the lateral
meniscus, the test is done with the knee internally rotated. Note that if there is an acute injury, the
above-mentioned test for meniscus tear is not valid because there is usually too much pain to flex the
knee very far. The sudden onset of decreased extension range of motion is a surgically significant sign

- 145 -
IOC Sport Medicine Manual 2000
of a displaced meniscal tear. If there is associated local tenderness over the meniscus and inability to
completely extend the knee, then there is a very good possibility of a displaced meniscal tear and
surgery should be considered on an urgent basis.

Visual examination of the knee joint should continue with the patient in the standing position. In the
weight-bearing position, the alignment of the knee extensor mechanism (femur, quadriceps femoris
muscle, patella, patellar tendon, and tibia) becomes apparent. The effects of foot position, and functional
leg length, on the knee joint become more evident in standing.

Observational assessment may also include a series of performance tests to determine the exercise
capacity and irritability of the knee. Observation of walking and running gait, hopping, stairs, a single
leg squat, or cutting manoeuvres can be used to determine the level at which the patient compensates
for knee dysfunction by altering their performance. The most common changes in knee joint function
during gait are decreased range of motion, decreased duration of the stance phase, and compensation
with altered hip and ankle motion.

Simple power tests are commonly used in place of more equipment-dependent strength tests to test
knee joint function. The one-legged hop for distance or one-legged hop for height, are two examples.
The person should take off and land on the same foot.
4 - Injuries to the Musculoskeletal
System

Figure 4.1 Patellar stability. Figure 4.2 Ranges of knee motion.

- 146 -
IOC Sport Medicine Manual 2000
Figure 4.3 Anterior drawer test.

Figure 4.4 Lachman test.

4 - Injuries to the Musculoskeletal


System

Figure 4.5 The pivot shift test involves flexing and extending the knee while applying a valgus strain.

- 147 -
IOC Sport Medicine Manual 2000
Figure 4.6 McMurray test.

Ankle and Foot


Examination of the ankle and foot should focus on the area of maximum tenderness, with careful
assessment of range of motion and function, attempting to isolate the various parts of the ankle and
foot as best as possible. It should be possible on physical examination to distinguish between a true
ankle problem versus a hind foot or subtalar joint problem. Distinguishing between the midfoot and
forefoot region, assessment of deformity in both the stance and swing phase of the gait, as well as
shoe wear patterns is important to the evaluation of the ankle and foot.

Elbow, Hand and Wrist


The same principles as described above apply in the examination of the distal upper extremity.
Correlating the point of maximal tenderness with the anatomy is critical in defining and confirming
the pathology. It is also important to understand the normal asymmetries that occur, particularly in
athletes involved in racquet or throwing sports where the dominant upper extremity will show
hypertrophy compared to the opposite side.
4 - Injuries to the Musculoskeletal

With respect to the elbow, the range of motion should be carefully documented. It is typical for intra-
articular pathology to lead to a loss of extension. Range of motion should include documentation of
supination and pronation. Examination of elbow stability can be difficult because of the need to control
rotation and forearm supination-pronation. A typical, but often subtle, instability occurs on the medial
System

side in throwing athletes. Application of a valgus force in full extension and subsequently in 15-20 of
flexion with the forearm pronated may allow comparison from side-to-side, and thus detection of the
subtle instabilities. Abnormal opening on valgus stress testing compared to the opposite side would
be suggestive of medial or ulnar collateral ligament problems. Resistance to supination-pronation and
wrist extension-flexion will help identify muscular tenderness problems in the elbow and forearm.

With respect to the wrist, the complexity of the carpal bones makes wrist examination confusing and
difficult. Care should be taken to systematically identify each carpal bone and compare it to the opposite
side with the intent of identifying the point of maximal tenderness and any deformity or asymmetry
that might be present. Assessing range of motion and stressing the carpals through distraction,
compression and rotation may elicit carpal instabilities or other areas of pain. Since one of the more
common injuries in sports is a scaphoid fracture, care must be taken to palpate the tubercle of the
scaphoid on the volar side and the region of the waist of the scaphoid through the anatomic snuff box.

- 148 -
IOC Sport Medicine Manual 2000
Careful examination and documentation of the neurovascular status of the hand and wrist are particularly
important in traumatic cases such as lacerations, crush injuries, or injuries suspicious of a fracture.
Documentation of the status of the median, radial and ulnar nerves is very important particularly at
the initial assessment of the injured athlete. This assessment becomes the baseline for any further
evaluations such as a post-operative assessment to ensure that the neurovascular status is intact.
Additionally, the individual tendons to the fingers (both flexors and extensors), should be assessed to
ensure that function is present. Avulsion tendon injuries are common, particularly in the fingers.
Puncture wounds may cause composite injuries to both nerves and tendons, making the assessment
even more confusing.

III. Case Histories

A. Shoulder

The shoulder is commonly injured in sport. With the advent of ultrasonography, MRI evaluation, and
arthroscopy, the accuracy of diagnostic entities in the shoulder has dramatically increased.
Unfortunately, so has the confusion with respect to the clinical evaluation and the overlap between
diagnoses and treatment options.

Shoulder Instability
Instability in the shoulder represents a complex spectrum of problems ranging from the obvious anterior
traumatic dislocation to the individual with excessively lax ligaments resulting in multi-directional
instability. The mechanisms of injury can be acutely traumatic, recurrently traumatic or related to
overuse. The following case histories are intended to demonstrate typical shoulder instability patterns.

1. Shoulder Dislocation
Within the example of an acutely traumatic dislocation, the mechanism can be one of indirect force on
the outstretched arm resulting in the humeral head coming forward through a lever action. Shoulder
dislocation can also occur as a result of a direct blow to the posterior aspect of the shoulder, literally
knocking the humeral head out of the socket or, as a result of a distraction injury with the arm being
pulled anteriorly out of the socket. In each case, the diagnosis may be an anterior dislocation but the

4 - Injuries to the Musculoskeletal


mechanism, although traumatic in each situation, is quite different.

Case History - Shoulder Dislocation

System
A 21-year-old ice hockey player was checked to the ice by an opposing player. As he fell, he
landed on his outstretched right arm. He immediately felt pain in the shoulder and was unable to
bring his arm back to his side. He felt diffuse tingling radiating down his right arm that soon
passed. He was aware of a popping sensation at the time of the injury but was not aware of any
major grating or grinding.

He was taken off the ice to a medical facility. Examination revealed a deformity in relation to
the general outline of his right shoulder with fullness in the anterior aspect of the shoulder, and
loss of the normal curvature of the lateral aspect of the shoulder. Neurovascular function in the
right upper extremity (with particular emphasis on the axillary nerve and distal pulses) was within
normal limits.

- 149 -
IOC Sport Medicine Manual 2000
Discussion
Differential diagnoses include:
a. acute anterior (anteroinferior)
traumatic dislocation right shoulder
b. fracture dislocation right shoulder
c. fracture proximal humerus right
shoulder

In ideal circumstances, an x-ray would be


available immediately to confirm the diagnosis.
In situations where the attending physician is
confident of the diagnosis of a dislocation, he
or she could reasonably proceed with a
reduction of the dislocation. This can be
performed in many ways but the principles of
reduction are to achieve relaxation in order to
overcome the typical muscle spasm, apply traction to the injured extremity (a gentle push on the
humeral head or rotation of the scapula), and reduce the shoulder as gently as possible.

There is a great deal of debate on whether shoulder dislocation should be reduced in the field. If the
injured athlete has a history of recurrent dislocations, a judicious attempt at reduction is quite reasonable.
In a situation of a first time dislocation, it is typically much easier to reduce the shoulder immediately
than waiting until the pain, inflammation and muscle spasm has increased. However, the decision to
reduce an acute dislocation should be based upon the experience and confidence of the attending
practitioner. Irrespective of how it is managed acutely (on the sideline, in the field or in a nearby
medical treatment room), careful documentation of the neurovascular status must be performed and
written down prior to and after any treatment. As noted above, an x-ray should be taken if at all
possible prior to reduction and subsequently following reduction. An axial view and a true anterior
view of the glenohumeral joint are the minimal acceptable views to elucidate the dislocation and any
associated bony injuries.

The principles of managing this injury are as follows:


4 - Injuries to the Musculoskeletal

 documentation of the injury


 reduction of the shoulder
 clinical and radiological confirmation of reduction
 identification of associated injuries to the bones, neurovascular and muscular tendinous
System

structures
 a period of immobilization until symptoms have subsided
 protection from re-injury
 rehabilitation prior to return to sport
The risk of recurrence following a first time traumatic dislocation is anywhere from 50-90% and is
dependent upon the type of sport and the shoulder dominance involved. Because of the high incidence
of recurrence of instability in the athletic population under the age of thirty, an acute surgical repair
should be considered. There is evidence in the literature from prospective case series and randomised
clinical trials to support acute surgical repair of a first time dislocated shoulder in a high-risk population.
Surgical management for traumatic anterior instability has a high success rate of about 90%.

- 150 -
IOC Sport Medicine Manual 2000
In the situation where the athlete in question has had a recurrent dislocation of their shoulder, the use
of a brace that prevents abduction and external rotation may be appropriate during the season. Shoulder
surgery should be considered at the end of the season.

There are many different techniques for surgical management that can be categorized as either
arthroscopic or open techniques. The principles of surgical management are to correct the pathology
that typically involves damage to the glenoid labrum or avulsion of the anteroinferior glenohumeral
ligament. Irrespective of the specific management, whether surgical or non-surgical, the rehabilitation
of the shoulder is critical to the long-term function and success with respect to return to sport.

In this example of the hockey player, the injured athlete was protected in a shoulder immobilizer for
two weeks until the symptoms had subsided. His initial rehabilitation programme consisted of gentle
movement exercise and gentle isometric strengthening. Although the athlete should not move the
shoulder into positions of instability, gentle active-assisted motion with bilateral pulleys improves
subjective discomfort and promotes returning function. Isometrics can generally be started very early
in the rehabilitation process if undertaken in a comfortable position.

As the hockey players pain-free range of motion and isometric strength improved, he was then
rehabilitated with a rotator cuff and scapular strength programme with associated proprioceptive
exercises. He was prevented from placing his arm in the provocative position of abduction and external
rotation for a period of six weeks post-injury. Range of motion during shoulder internal/external
rotation strength exercises was limited to about 10o of external rotation. Abduction was also limited to
the first 45o. Isometric exercises can be gradually extended into shoulder flexion to encourage muscle
contraction that would stabilize the shoulder.

Once range of motion was pain-free throughout his shoulder movement, and he could strongly resist
forces applied to the arm in a variety of positions, the hockey player was placed in a protective brace
and allowed to return to play for the remainder of the season.

At the end of the season he was advised to undergo surgical repair.

4 - Injuries to the Musculoskeletal


2. Recurrent Subluxation Of The Shoulder

Case History - Recurrent Subluxation of the Shoulder

System
A 23-year-old volleyball player was attempting a block during a match and the spiked ball hit
the palm of his hand forcing his arm beyond the normal range of motion, causing pain in the front
of his shoulder. He shook it off initially and continued playing. After the game he noticed some
pain in his shoulder and iced the shoulder. The following day, the shoulder was more painful. The
mechanism of injury was clearly elucidated and the player was asked whether he felt that his
shoulder came out of place or not. Although the athlete could not be sure, it was suspected that the
shoulder did not dislocate during the injury. He was therefore treated with ice, anti-inflammatory
medications to reduce the pain and inflammation. Rehabilitative exercises were initiated as tolerated
by pain. Since strengthening is considered to be specific to the mode of contraction, exercises
followed a typical progression: isometric, isotonic, isokinetic, and finally eccentric modes of
contraction. Eccentric exercises mimic the acceleration-deceleration sequence that occurs with
volleyball serves and spikes. Shoulder strengthening is also specific to the position of the shoulder.
Therefore, isometric exercises are initiated at 20 o of shoulder flexion, and then progressed to

- 151 -
IOC Sport Medicine Manual 2000
isometric exercises at 170 o of shoulder flexion to mimic the position of a volleyball block. Weight-
bearing exercises, like one-arm wall push-ups, also assist in developing shoulder girdle and shoulder
stability in a flexion position.

He was allowed to return to play when he was asymptomatic and his strength was within
normal limits.

During the remainder of the volleyball season he noticed three similar episodes. One occurred
when he went to dig a ball off the court and landed on his outstretched arm. Another episode
occurred when he was forced to spike a ball with his non-dominant arm, and the third episode
occurred in a similar way when he was making a block. With each episode, he had acute pain and
aching for a period of 2-3 days. He was able to continue playing because this was his non-dominant
shoulder but he continued to be symptomatic. He was seen at the end of the season and evaluated
by x-ray that was entirely normal. He was symptomatic despite aggressive rehabilitation. He was
felt to have a diagnosis of recurrent anteroinferior subluxation based upon the mechanisms of
injury and the recurrent symptoms. He was referred to a surgeon who subsequently performed
arthroscopy and repair of his anteroinferior labral tear that was identified at the time of arthroscopy.

Discussion
This example represents a recurrent subluxation of the shoulder.
It involves the non-dominant shoulder and although the history
was consistent with an instability episode, there was never any
clinical or other evidence of a dislocation. One would expect
that compared to a dislocating shoulder, this diagnosis would
have a better prognosis. However, the prognosis in somebody
with recurrent episodes of subluxation is very similar to an
individual with recurrent episodes of dislocation. The treatment
of choice is typically surgical. Since this is the athletes non-
dominant arm, the surgical approach could be either arthroscopic
or open. Concerns about loss of range of motion in the non-
dominant arm because of an open procedure are not as significant
as they would be if this were his hitting arm.
4 - Injuries to the Musculoskeletal

People with recurrent subluxation may describe a clinical


syndrome of recurrent dead arm symptoms. At the time of the
subluxation the arm feels as though it is dead, there might be
some tingling and pain, but after a few minutes the athlete is
System

able to shake it off and continue playing.

It should be pointed out that the same symptoms described in recurrent subluxations could occur from
a non-traumatic injury mechanism as a result of overuse, but this usually occurs in the dominant
shoulder. This syndrome will be discussed in the next case history.

3. The Overhead Athletes Shoulder


Sports such as swimming, volleyball, tennis, gymnastics, and in particular, baseball throwing, place a
great deal of stress on the shoulders. These repetitive stresses are compounded by the fact that the
normal imbalance between internal and external rotation strength is magnified by repetitive use. An
additional factor in the young overhead athlete shoulder is inherent flexibility. It is common to see
young, immature athletes excelling in sports such as swimming and baseball throwing because their
flexibility allows them greater external rotation and therefore a greater leverage to perform the overhead

- 152 -
IOC Sport Medicine Manual 2000
maneuvers. Although this inherent flexibility is a normal phenomenon, problems can occur with
repetitive use and magnification of the imbalance in the shoulder muscles.

The two most common complaints in the overhead athlete are pain and loss of function. It is not usual
for the individual to complain of instability in the shoulder. It is important to identify the phase of the
overhead motion where the symptoms occur. Sometimes athletes will volunteer this information on
very specific questioning. For example, a baseball pitcher or outfielder may describe his arm symptoms
in the cocking phase or following the ball towards the target after release of the ball where they will
describe pain and occasionally a click. In a swimmer, the problems more often occur during the
butterfly stroke where there is no opportunity to use body roll to compensate for shoulder
hyperabduction and external rotation. It is important to ask questions about their performance, as it is
usually the case that performance times, velocity, endurance and accuracy are affected by these
problems.

The physical examination of the uni-dominant overhead athlete will typically demonstrate increased
strength on internal rotator muscles compared to relatively weak external rotators. There may be
weakness of the supraspinatus muscle as measured by elevation in the scapular plane. Often the
examiner will elicit signs of impingement with the arm in a forced forward flexed position or in
flexion internal rotation or possibly with a painful arc of motion from 60-120. There usually will be
a discrepancy between internal rotation range of motion when comparing one shoulder to the other,
with loss of internal rotation range of motion in the dominant shoulder and an increase of external
rotation in the abducted position of the dominant shoulder. Although this may be considered a normal
adaptive change in the range of motion and potentially an advantage during sports participation, when
the shoulder becomes problematic, this clearly is a maladaptive situation.

It is unlikely that true apprehension will be demonstrated in these athletes, but a positive relocation
sign may be present. A positive relocation sign is elicited with the patient lying supine on the examining
table. The affected arm is passively abducted to 90 and externally rotated until symptoms occur. The
arm is then internally rotated back to neutral. Then, the examiner places his opposite hand on the head
of the humerus with a posteriorly directed force. The arm is then externally rotated to the maximal
position. The test is considered positive when external rotation is improved with the posteriorly directed

4 - Injuries to the Musculoskeletal


force and symptoms are relieved. Upon release of the stabilizing hand, symptoms usually recur more
obviously and the patient shows immediate distress. A shoulder relocation test can be augmented by
placing the hand behind the humerus, but this is usually not necessary. The affected side can be compared
to the opposite side to ensure that it is positive only in the affected shoulder. However, in swimmers or

System
other overhead athletes that use both shoulders, it may be difficult to elicit any difference between the
two sides.

- 153 -
IOC Sport Medicine Manual 2000
Fig 4.7 Shoulder relocation test. Figure 4.8 Shoulder apprehension test.

Fig 4.9 A SLAP lesion test


a) pain with isometric resisted shoulder flexion while shoulder internally rotates
b) no pain with isometric resisted shoulder flexion while shoulder externally rotates.

The treatment of the overhead athletes shoulder should primarily involve rehabilitation. The main
focus of rehabilitation should be to correct the imbalances in the range of motion, strength, and
proprioceptive control of the shoulder itself. It is usually necessary to use some form of pain relief in
4 - Injuries to the Musculoskeletal

order for the rehabilitation to be effective. Anti-inflammatory medications by mouth are usually given
and occasionally a corticosteroid injection into the subacromial space is warranted. However, no form
of medication will substitute for rehabilitation. The medication should be used primarily to allow the
rehabilitation to take place.
System

The athlete will need to stop the sporting activity that is causing the problem for a period of time, and
use cross-training methods and other means to maintain his/her performance. It is also important to
consider the technique of the overhead activity to ensure that optimal technique is utilized in the
recovery process.

If the problem is seen immediately, a short course of rehabilitation may result in full return to activity.
However, most of these athletes work through their pain and problems for a period of time before it
comes to the attention of a physician. In these situations, effective rehabilitation and return to sport
may take a number of weeks or even months.

In situations where rehabilitation has failed, an arthroscopic evaluation and treatment is usually required.
With arthroscopy, the typical findings are perceptively increased laxity in the affected shoulder
compared to the opposite side, labral pathology either anteriorly or superiorly involving the bicep
- 154 -
IOC Sport Medicine Manual 2000
tendon attachment (SLAP lesion) and partial thickness humeral surface rotator cuff tears. Any
combinations of these arthroscopic findings can be seen in the overhead athlete. The arthroscopic
treatment involves repairing damaged labral tissue and correction of capsular laxity if present. Rotator
cuff pathology is repaired if it is approaching a full thickness lesion, otherwise it is debrided and it
should recover with the rehabilitation. Following this type of surgical repair, it will usually take up to
six months before the athlete returns to sport.

Case History - Overhead Athletes Shoulder

An 18-year-old male baseball pitcher presented with recurrent pain in his right, dominant
shoulder. Two seasons ago he noticed some pain towards the end of each game, particularly if he
had thrown more than one hundred pitches. Towards the end of the season he was getting tired
more easily, he was losing his control late in the game, and had to be relieved earlier then at the
beginning of the season.

He recovered in the off-season and was performing strengthening exercises, including heavy
bench pressing as he was trying to compensate for the problems he was having. In the following
season, he began having trouble early on with poor control and lower velocity. He eventually got
to the point where his coach took him out of the starting rotation and had him pitch middle inning
relief. His shoulder eventually got to the point where it was too painful to keep pitching and he
subsequently sought medical attention.

Discussion
Differential diagnoses include:
a. throwers shoulder
b. recurrent anterior subluxation
c. rotator cuff tear
d. supraspinal nerve injury
e. internal impingement

At the time of the initial evaluation, the shoulder was quite painful and he had signs of impingement.
There was weakness of external rotation and elevation in the scapular plane. He had a positive relocation
test and he demonstrated the typical findings of increased external rotation and loss of internal rotation

4 - Injuries to the Musculoskeletal


compared to the opposite shoulder. His actual range of motion in total was normal. The assumed
diagnosis in this athlete is throwers shoulder, which is a non-specific diagnosis that pathologically
can include partial thickness rotator cuff tears, subtle instability, superior labral tears, or combinations
of these lesions.

System
He was placed on some anti-inflammatory medications to reduce the pain and impingement symptoms.
He was treated with a specific rotator cuff strengthening programme that began with rotational exercises
at the side and eventually progressed to the point where he was doing strengthening exercises in the
throwing motion position. Electrical muscle stimulation can be used over the scapular muscles as an
adjuct to resisted exercise. He was also taught some proprioceptive exercises that included an endurance
component. For example, the athlete can bounce a basketball off a nearby wall, throwing and catching
with the same arm. Gradually the athlete should return to throwing, but a maintenance programme
for the stabilizing muscles is essential to continued athletic performance. He was unable to return to
play during that season.

Three months later, in the off-season, he was re-evaluated. The shoulder no longer demonstrated signs
of impingement. The strength and rotational imbalances had been corrected. His pitching coach was
brought into the discussion and issues of technique and utilization of lower extremity and trunk strength

- 155 -
IOC Sport Medicine Manual 2000
were emphasized. In the off-season he worked on an ongoing strengthening, stretching and rehabilitative
programme as well as technique fine-tuning. He returned to play the following season and was been
able to maintain his pitching velocity and control throughout.

The same scenario might have resulted in failure to return to play, at which time an arthroscopy would
be considered. A MRI or ultrasound is sometimes helpful in these situations to sort out the specific
pathology, but it is absolutely critical that clear and direct communication between the treating physician
or surgeon and the radiologist occurs. In situations where the communication is not available or the
radiologist is not experienced in reading a shoulder MRI, the MRI is at best confusing because of false
positive or false negative results.

4. Muscle-Tendon Injuries: The Shoulder Impingement Tendinopathy


The whole concept of impingement with respect to the athletes shoulder has changed in the past ten
years. The spectrum of impingement, from inflammation to partial damage to complete rotator cuff
tear related to a narrowing of the space between the humeral head and the coracoacromial arch, does
not necessarily apply in the injured athlete. The phenomenon of impingement is more often a dynamic
4 - Injuries to the Musculoskeletal

or secondary phenomenon. In the athlete, commonly there is a loss of muscular control of the humeral
head. The humerus translates superiorly and squeezes the tendon structures between it and the
coracoacromial arch, rather than a specific narrowing of the anatomical space. More recently, the
concept of internal impingement has been demonstrated. This occurs when the arm is abducted and at
System

extremes of external rotation of the humeral head and posterosuperior glenoid abut against each other,
pinching the infraspinatus tendon. This type of impingement is usually only seen in athletes participating
in overhead sports, and is related to excessive external rotation and capsular lesions.

In the current case it is more appropriate to think of the tendon structures surrounding the shoulder as
stabilizers and therefore failure of this synergistic or coordinated function will result in subtle instability
as detailed in the overhead athletes shoulder in the previous section. The tendons can also fail as a
result of overuse or acute traumatic events. The failure can result in intrinsic tendinopathy, partial or
full thickness tearing.

- 156 -
IOC Sport Medicine Manual 2000
Case History - Shoulder Impingement Tendinopathy

A 53-year-old gentleman, an avid right-handed golfer, was about to play his ball deep in the
rough 185 yards from the green. He attempted to go for the green with a 4 iron but missed the ball
cleanly and took a large divot out of the ground. He noticed immediate pain in his left, lead shoulder.
He continued to play the remainder of the round, but the pain got progressively worse to the point
where he could no longer play. That evening, the pain was worse. He had difficulty raising his arm
and after a few days sought medical attention.

At the time of his initial evaluation, he was complaining of pain in the anterosuperior aspect of
his left shoulder. He had pain at night and difficulty sleeping on that side. He denied any previous
shoulder problems, although he had some aching pain on and off for a few years which was of no
major consequence. He used anti-inflammatories in the past and obtained benefit. He was otherwise
healthy.

His examination revealed no obvious abnormality. There was tenderness at the anterosuperior
aspect of his shoulder overlying the rotator cuff. His range of motion was within normal limits
although he had difficulty raising his arm in the scapular plane. This resulted in an abnormal
scapular humeral rhythm and he clearly had pain as his shoulder was abducted through an arc
between 60-120. There was weakness of elevation in the scapular plane with minimal pain-related
weakness of external rotation. Internal rotation was normal. Translation tests were within normal
limits. He demonstrated all of the signs of impingement. His neck examination and distal
neurovascular status was entirely normal.

Discussion
Differential diagnoses include:
a. rotator cuff tear
b. rotator cuff tendinopathy
c. rotator cuff calcific tendinopathy

There are many terms that relate to the symptoms and signs in this gentleman including rotator cuff
tendonitis, bursitis, tendinopathy, and rotator cuff tear. It is important to recognize that this golfer

4 - Injuries to the Musculoskeletal


probably had some age-related and overuse-related tendon changes and that the acute injury resulted
in a tear of the rotator cuff, usually the supraspinatus tendon. A subacromial injection of local anaesthetic
will typically result in relief of the pain and the impingement signs. However, in cases where the
rotator cuff tendon has partially or fully ruptured, the weakness will be present. It will be possible to

System
distinguish true weakness from pain related weakness following a subacromial injection of local
anaesthetic.

This person should be investigated to identify whether or not a full thickness rotator cuff tear can be
identified. An arteriogram, MRI, or ultrasound will usually confirm the pathology.

In this situation, corticosteroids do not have a role to play and an aggressive approach is indicated. A
surgical repair of the rotator cuff is indicated, assuming the patient is otherwise healthy. Surgically
repairing the tendon will likely prevent the chronic retraction of the muscle tendon unit and the atrophy
and loss of function that might otherwise occur. It will take six months to recover from this type of
surgery but the results of surgery are usually very good, with full return to golf.

The same scenario can occur without a rotator cuff tear, with simply an injury to the rotator cuff. This
would require a period of activity modification, the use of non-steroidal anti-inflammatories and

- 157 -
IOC Sport Medicine Manual 2000
possibly a corticosteroid injection to relieve pain followed by rehabilitation of the rotator cuff. It may
be difficult to distinguish a rotator cuff tear from rotator cuff tendinopathy or tendinitis. The subacromial
injection of Xylocaine is often quite helpful particularly when the pain is relieved and strength is
normal. This would indicate that there is no significant tearing of the rotator cuff. Full recovery requires
rehabilitation and strengthening of the rotator cuff. Failure to rehabilitate the rotator cuff will result in
a poor outcome.

5. The Acromioclavicular Joint


The acromioclavicular joint is commonly involved in sport-related injuries. There are three clinical
diagnoses that occur most commonly and each will be illustrated with an example.

Case History - Acromioclavicular Joint Dislocation

A 25-year-old soccer player jumped to head a ball. He was hit by the opposing goalkeeper, lost
his balance in mid air and landed on the point of his dominant right shoulder. He had immediate
pain and felt a tearing sensation in the top of his shoulder. He was unable to continue playing. He
was seen on the sideline by the trainer and had an obvious deformity in the region of the AC joint.
He was sent for further evaluation to his physician who noted that he had a deformity involving
the AC joint, with prominence of the distal clavicle. X-ray was performed to rule out a fracture.
The clavicle and the acromion were no longer opposed, indicating a dislocation at the
acromioclavicular joint.

Discussion
Differential diagnoses include:
a. AC joint subluxation/sprain
b. AC joint dislocation
c. fractured clavicle

X-rays that are typically done for AC joint problems are done with weights and without weights in
one plane only, but this does not adequately evaluate the joint from a radiological standpoint. An x-
ray must be taken at 90 to the anterior-posterior plane. An axillary view will evaluate the AC joint at
90 to the typical films and assess whether or not the clavicle is displaced in a posterior direction with
respect to the acromion. Comparable weight-bearing views of the uninjured shoulder should be
4 - Injuries to the Musculoskeletal

performed.

AC joint injuries have been classified into three types. Type one is simply a minor sprain of the
acromioclavicular ligaments. Type two is where there is subluxation of the joint with partial damage
System

to the conoid and trapezoid ligaments. Type three is where there is complete disruption of all the
ligament structures and dislocation of the joint.

Non-operative treatment should involve protection for a period


of time in a shoulder immobilizer or equivalent until the athlete
is asymptomatic. Protection of the upper extremity should be
followed by rehabilitation and gradual return to sport, similar
to the programmes described above. Cross adduction should be
avoided until it is pain-free. Modification to equipment or
fabrication of a custom-made splint for the shoulder girdle can
offer the athlete some protection during participation as they
are recovering. Return to function is the norm with non-operative
treatment. Occasionally there will be long-term pain at the region
of the AC joint or problems with instability. If this is the case,
- 158 -
IOC Sport Medicine Manual 2000
delayed surgical resection of the clavicle with reconstruction of the joint can be considered. Surgical
treatment of these injuries may involve a variety of different techniques, all of which have been
associated with significant complications.

Case History - Acromioclavicular Joint Osteolysis

A 30-year-old weightlifter presented with progressive increasing pain on the superior aspect of
his dominant shoulder. He has been a weightlifter for five years. He denied any specific injury but
has gradually been getting to the point where he could not do his normal routine. He had some
trouble doing a bench press but there was more trouble with the military press and pulls down
behind his head. He described pain of an aching nature that was occasionally sharp on the top of
his shoulder. He admitted difficulties sleeping. His examination isolated the area of maximal
tenderness to the AC joint at the end of the clavicle. The rest of the examination was unremarkable.

X-ray examination revealed some very minor changes in the distal clavicle. A bone scan was
ordered that showed increased uptake in the region of the distal clavicle.

Discussion:
Differential diagnoses include:
a. distal clavicular osteolysis
b. AC joint arthrosis
c. rotator cuff tendinopathy

The x-ray examination showed erosion (osteolysis) at the end of


the clavicle. The x-ray findings were not as obvious as some, but
bone scan confirmation was helpful. This was a classic case of
distal clavicular osteolysis. This condition may occur following
direct trauma such as a grade 1 acromioclavicular joint sprain or
repeated micro-trauma as with weight training.

The treatment of this condition was to use rest, ice, and anti-
inflammatory medication.

4 - Injuries to the Musculoskeletal


Corticosteroids are contra-indicated because of increasing the
osteolytic process. Modification of weight lifting technique is
essential to avoid all weight training exercises with the shoulder

System
elevated or abducted above 90o. Time to resolution can be 3-12
months. If the problem is recalcitrant to non-operative means, or
the athlete is unable to rest for the necessary period of time, surgical
resection of the outer end of the clavicle can be performed
arthroscopically or through a small open incision. The results seem
to be equally as effective.

- 159 -
IOC Sport Medicine Manual 2000
Case History - Acromioclavicular Joint Arthritis

A 40-year-old healthy right hand dominant tennis player presented with pain in her right anterior
shoulder. She noticed the pain getting worse when she was working in her garden on the weekend.
When she returned to playing tennis during the week, she noticed ongoing pain in her shoulder.
This was worse after she played and towards the evening. She described an aching pain on the top
of her shoulder. There were no obvious associated symptoms. This problem progressed for a number
of weeks before she sought medical attention.

Her physician initially assessed her. There was no evidence of any problems related to her neck
or loss of range of motion in the shoulder. There was pain when the arm was placed across her
chest with cross arm adduction. The pain was localized to the AC joint. There was a painful arc
from 120-180 in the coronal plane. Strength testing and the remainder of the examination were
unremarkable except for tenderness in the AC joint. X-ray was taken that revealed degenerative
arthritic changes in the AC joint.

Discussion
Differential diagnoses include:
a. AC joint arthrosis/arthritis
b. distal clavicular osteolysis
c. rotator cuff tendinopathy

The problem of AC joint arthritis occurs quite commonly in people


in their forties and fifties, although it is usually an asymptomatic
finding on x-ray. In situations where AC joint arthritis has become
symptomatic, the usual non-operative treatment measures for arthritis
should be considered, such as local treatment with ice after activity,
careful warm up before activity, stretching and maintenance of strength
around the area. The use of anti-inflammatories may be helpful and
corticosteroid injections into the AC joint can significantly improve
the problem. In situations that are recalcitrant non-operative treatment,
surgical resection of the end of the clavicle can be an effective treatment.
4 - Injuries to the Musculoskeletal

6. Fractures
Fractures in the shoulder girdle are quite common in younger people including skeletally immature
individuals. There is also a peak incidence of fractures in the more elderly population but these are
System

typically not related to sport. The following case history will demonstrate these injuries.

Case History - Fractures in the Shoulder Girdle

A healthy 22-year-old male was playing ice hockey and was checked from the side jamming his
shoulder into the boards. He had immediate pain and was unable to continue playing. He was
seen and assessed in the locker room, at which time he had significant tenderness in the region of
the middle third of his clavicle. There was an associated deformity with obvious disruption of his
clavicle. He was sent for x-ray that revealed a minimally comminuted fracture of the mid-shaft of
the clavicle.

He was treated in a figure of eight sling for a period of 3-4 weeks to attempt to maintain the
length of the clavicle and apposition of the bony fragments. At four weeks, repeat x-ray revealed
some early healing. He was advised to avoid contact for twelve weeks.

- 160 -
IOC Sport Medicine Manual 2000
Discussion
Differential diagnoses include:
a. fractured clavicle
b. AC joint dislocation

Clavicle fractures are very common in younger individuals. They usually heal without any major
problems provided that there is avoidance of re-injury. It is quite common for young athletes to feel as
though they are ready to return to sport as early as four weeks after the injury. However, it will take at
least three months for bony union to be complete and solid. It the meantime, it is important to ensure
that there is a full rehabilitation programme to maintain range of motion, and restore strength and
endurance during the recovery period.

Sometimes the fracture is quite comminuted and spikes of bone will tense the skin overlying the
fracture. In these situations, using a securely applied figure of eight splint will help reduce the tension
on the overlying skin. Rarely an open reduction is required if the fracture is open. In the professional
athlete, and the ready availability of orthopaedic sports surgeons, open reduction and internal fixation
by plating the fracture will permit resumption of contact sport in 3-4 weeks. The plate should be
removed at the end of the season.

Clavicle fractures near the AC joint have a higher incidence of non-union particularly if the fracture
occurs just medial to the coracoclavicular ligaments. In this situation, the entire weight of the arm
tends to distract the fracture and can result in a malunion, delayed, or non-union. Surgical fixation
should be considered in this situation if the reduction cannot be maintained adequately with a shoulder
immobilizer.

Other fractures around the shoulder in athletes are relatively uncommon. In children, proximal humeral
fractures are common and can usually be managed non-operatively. They heal rapidly and do not
result in any long-term problems in most cases. Fractures associated with dislocations typically involve
the greater tuberosity of the humerus. Provided the tuberosity is reduced adequately, surgery is not
indicated. If there is significant displacement, typically greater than 1 cm, surgical reduction and
fixation of the fracture should be considered.

4 - Injuries to the Musculoskeletal


B. Elbow

1. Acute Tennis Elbow Syndrome

System
Case History - Acute Tennis Elbow Syndrome

A 40-year-old recreational tennis player felt a sudden onset of pain in the lateral aspect of her
right elbow as she struck the ball during a backhand volley. She continued to play, but approximately
one hour later, she noted increased pain over the lateral aspect of the right elbow with discomfort
on attempts to dorsiflex the wrist. This pain had occurred on several occasions in the past six
weeks.

Specific clinical examination revealed some minor swelling over the lateral epicondylar region
with marked point tenderness. There was pain on resisted wrist extension. The elbow demonstrated
a full range of motion and there was full, pain-free cervical and shoulder range of motion. There
was no neurovascular deficit in the distal limb. X-ray of the area showed no bony abnormalities.

- 161 -
IOC Sport Medicine Manual 2000
Discussion
Differential diagnoses include:
a. loose body within the elbow joint
b. radiocapitellar degeneration
c. posterior interosseous nerve entrapment
d. thoracic outlet syndrome
e. cervical spondylosis of C6 with root irritation
f. acute lateral epicondylitis

Careful clinical examination would attempt to diagnose whether the pain was coming from the lateral
humeroepicondylar region, the region of the capitellum or from the radial head itself. Palpation of any
crepitus within the joint would give a clue to possible radiocapitellar degeneration.

Careful neurological examination should be carried out to ascertain whether there is any altered
sensation in specific dermatomes (as in the C6 distribution) or any alteration in peripheral nerve
distribution. Posterior interosseous nerve entrapment would give rise to marked weakness, particularly
of the digital extensors, but no sensory change.

X-ray of the elbow should demonstrate any intra-articular loose bodies, previous radiocapitellar
degeneration or changes within the joint or lateral humeral epicondyle. The cervical spine should be
x-rayed if one suspects the pain originates from this source.

If the elbow pain is found to be due to loose bodies within the elbow, these can often be removed by
arthroscopy.

Upon finding point tenderness over the lateral epicondyle with pain radiating into the common extensor
origin in the absence of other changes, the diagnosis of acute tennis elbow syndrome was made.

Initial management consisted of rest until the pain had eased, local ice massage three times per day,
and anti-inflammatories.
4 - Injuries to the Musculoskeletal
System

Figure 4.10 Tennis elbow.


- 162 -
IOC Sport Medicine Manual 2000
Modalities including pulsed low dose ultrasound, interferential therapy and electric muscle stimulation
were used in the initial stage of treatment. Acute pain can be reduced by wrist braces or taping. Active
ROM and isometric exercises of the wrist and elbow began at 1 week post-injury. As the tennis players
elbow pain began to subside, strengthening was progressed to include concentric and eccentric wrist
extension exercises (Figure 4.18) and grip strengthening exercises with a wet sponge. Wrist and forearm
stretches were introduced to the programme as her range of motion improved and pain resolved (Figure
4.11). Her cardiovascular system was maintained during the initial stages of treatment through a
cycling programme.

Figure 4.11 a) Right wrist flexor and b) extensor stretches.

The tennis player was returned to sport-specific exercise including backhand and forehand drills at
three weeks post-injury. Proprioception exercise was started which included a progression of upper
extremity weight-bearing exercises. Her wrist was taped initially to reduce stress on the common
extensor origin as she was returning to sport. Her backhand stroke was corrected so that she generated
more power from her lower body and trunk and the string tension of her racquet was adjusted. Her
grip size was assessed by measuring from the proximal palmar wrist crease to the tip of the ring finger

4 - Injuries to the Musculoskeletal


and found to be appropriate. This athlete responded well to conservative treatment and was returned
to game conditions at 6 weeks post-injury with a tennis elbow strap.

If symptoms had persisted and alternate sources of lateral elbow pain have been ruled out, a local

System
corticosteroid injection may be considered. This injection may be repeated up to three times at two-
week intervals. In extreme cases of lateral epicondylitis, surgical management may be required.

- 163 -
IOC Sport Medicine Manual 2000
2. Elbow Dislocation

Case History - Elbow Dislocation

A 24-year-old soccer player was tripped as he tried to run between two defenders. He landed
forward on an outstretched hand. He was immediately aware of marked pain in the elbow joint
with immediate onset of swelling. He was unable to actively flex and extend the elbow because of
pain. Initially, he was aware of minor diffuse altered sensation in his hand. This later became
more distinct, with decreasing sensation to touch and pin prick on the ulnar border of the hand.
There was no major skin lesion in relation to the elbow.

The arm was initially splinted and the patient was removed from the field to a treatment facility.
Inspection of the elbow revealed an obvious deformity with marked swelling. There was marked
crepitus on attempted motion of the elbow joint, which remained extremely painful. It appeared
that there had been a fracture. Neurovascular examination revealed some minor decreased sensation
in the ulnar nerve distribution but the medial and radial nerves appeared normal. The distal
radial pulse was normal.

Discussion
Differential diagnoses include:
a. fractured olecranon
b. dislocation or fracture-dislocation of the elbow joint
c. condylar fracture of the distal humerus
d. supracondylar fracture of the humerus
e. Galeazzi fracture dislocation of the elbow
f. fractured head of radius

Before x-rays are taken, it is important that the elbow is splinted in a position of maximum comfort.
It is important to continue to monitor neurovascular function to the distal limb to ensure it is not
further compromised. X-ray examination would allow visualization of all of the bones of the elbow
joint. Dislocations should be reduced following x-ray examination.

Any fracture that has occurred around the elbow joint, particularly in a young athlete, usually requires
4 - Injuries to the Musculoskeletal

open reduction and internal fixation to ensure adequate restoration of the bony anatomy to minimize
subsequent osteoarthritis. It is important to examine the bony anatomy of the epicondyles of the distal
humerus, the olecranon, the radial head, the superior radio-ulnar joint, and the radiohumeral joint.
Elbow dislocations without associated fractures can often be managed without an operation.
System

Following a fracture or dislocation to the elbow joint, it is important to regain range of motion as soon
as possible. Failure to do so may result in a fixed flexion deformity and possible loss of full flexion in
the joint because of soft tissue scarring around the elbow, and/or heterotopic ossification. Regaining
full range of motion is extremely uncommon following significant injury to the elbow joint.

A common complication of repeated minor trauma to the elbow is that of an olecranon bursitis that
presents as a recurrent swelling over the point of the elbow. In a somewhat quiescent phase, one often
gets the impression of loose bodies within the olecranon bursa. The olecranon bursa may swell
considerably to the size of a golf ball or even larger. If this occurs, it may require aspiration. If it
becomes recurrent, excision of the olecranon bursa may be recommended.

- 164 -
IOC Sport Medicine Manual 2000
Figure 4.12 Fractured olecranon.

Based on clinical and radiological findings, this soccer player was diagnosed with an elbow fracture-
dislocation. He was admitted to hospital where he underwent an open reduction and internal fixation
of the radial head. Once the wound had healed, he was allowed to return to running, cycling and lower
extremity strength training. The elbow was immobilized in a plaster cast for 1 week, which was
replaced with a hinged elbow orthosis with an extension block where the elbow was determined to be
unstable. He was allowed to remove the orthosis for daily physiotherapy that consisted of contrast
baths and interferential therapy. He was instructed in active flexion and extension range of motion
exercises out of the orthosis and was instructed to limit his extension to his pain-free range of motion.
Passive mobilization to restore extension was also included in the treatment programme. As the stability
of the joint improved, the extension block was decreased by 10 per week. Isometric exercises of the
wrist (Figure 4.13) and elbow (Figure 4.14) were begun. Cardiovascular fitness was maintained

4 - Injuries to the Musculoskeletal


throughout this period through a cycling and lower body weight training programme.

At 6 weeks post-injury, the soccer player was advised to discontinue use of the orthosis and encouraged
to begin biceps, triceps (Figure 4.15), and forearm stretches (Figure 4.11). Strengthening was progressed

System
to include concentric and eccentric exercises of the elbow and wrist.

- 165 -
IOC Sport Medicine Manual 2000
Figure 4.13 Isometric exercises of right wrist (arrow showing force applied against resistance).

Figure 4.14 Isometric exercises of left elbow (arrow showing force applied against resistance).
4 - Injuries to the Musculoskeletal
System

Figure 4.15 Triceps stretch.

Proprioception retraining was introduced at 6 weeks post-injury which included upper limb weight-
bearing exercises and other sport specific drills including running, throwing, passing and kicking.
This soccer player was allowed to return to full activity at 8 weeks post-injury.

- 166 -
IOC Sport Medicine Manual 2000
3. Throwers Elbow

Case History - Throwers Elbow

A 29-year-old pitcher presented with a 3 month history of medial elbow pain when throwing.
There was no specific injury. He also experienced paraesthesia in his ulnar 2 digits when throwing.

Examination showed a flexion contracture of 10, full flexion, pronation and supination and
pain medially on terminal extension. Tinels sign was positive for the ulnar nerve at the elbow, but
there were no motor or sensory deficits. Valgus stress elicited pain medially near extension, but
there was no obvious instability.

Discussion
Differential diagnoses include:
a. valgus extension overload
b. medial collateral ligament tear
c. common flexor-pronator tendon origin tear
d. ulnar nerve entrapment
e. thoracic outlet syndrome
f. cervical radiculopathy (C8)

Whereas tennis elbow classically occurs in relation to the lateral humeral epicondyle, similar symptoms
may occur in the medial humeral epicondyle. This is termed throwers elbow and may also occur in
javelin throwers, tennis players and athletes involved in throwing sports. This condition occurs as the
arm is whipped first backwards and then forwards in the throwing motion, resulting in an abnormal
valgus stress at the common flexor origin from the medial epicondyle, the medial collateral ligament,
and/or ulnar nerve.

Treatment for medial epicondylar problems is very similar to that employed in lateral epicondylar
problems, with the addition of modification of throwing technique which is very important in the
treatment programme. Medial collateral ligament tears would also be treated with rest, ice, anti-
inflammatories and modification of throwing technique. In some cases, ligament reconstruction is

4 - Injuries to the Musculoskeletal


necessary.

Special care must be paid to the presence of the ulnar nerve behind the medial humeral epicondyle.
The superficial position of the ulnar nerve can result in irritation and cause symptoms of weakness in

System
the ulnar deep flexors, and altered sensation along the ulnar border of the hand, the fifth digit, and part
of the fourth digit. A positive Tinels sign may be elicited and the ulnar nerve may be found to be
subluxing out of its groove with flexion.

The ulnar nerve may be entrapped at the elbow. It may also be entrapped in Guyons canal in the wrist,
particularly in anyone who puts pressure on the dorsiflexed wrist, such as gymnasts and cyclists.
Once the site of entrapment has been identified, this can often be treated with splinting. Anti-
inflammatories can help reduce local edema. If this does not prove satisfactory, surgical exploration
and neurolysis may be required.

- 167 -
IOC Sport Medicine Manual 2000
Figure 4.16 Results of repeated valgus stress to elbow.

Valgus extension overload refers to a condition where there is impingement of structures in the olecranon
fossa of the distal humerus as the elbow is extended. The impingement can occur when the olecranon
tip strikes the olecranon fossa. This can be caused by osteophytes on the olecranon and/or in the
olecranon fossa. Hypertrophy of soft tissue structures (ie. joint capsule, fat pad) can result in
impingement of these structures between the olecranon and the olecranon fossa during terminal
extension. CT scan and/or MRI can be used to define potential structures included in valgus extension
overload.

This 29-year-old pitcher was diagnosed with valgus extension overload syndrome and was treated
with anti-inflammatories, rest and ice. He was referred to a physiotherapist for treatment that consisted
initially of electrical modalities including ultrasound, laser and interferential current. Active ROM
and isometric exercises of the wrist and elbow were started immediately. Stretching of the biceps,
triceps, common flexor pronator and common extensor origins were started 1 week later. Rotator cuff
strengthening was started immediately and concentric and eccentric elbow and wrist exercises were
4 - Injuries to the Musculoskeletal

included at 3 weeks post-injury. When he was able to weight-bear fully through the upper limbs,
proprioception exercises were begun which included wall push ups, chin ups and upper limb weight-
bearing exercises with the physiotherapy ball. He was allowed to return to sport activity at 6 weeks
post-injury by performing light throwing motions without the ball. He then progressed to throwing
System

around the infield and finally to longer distance throws to the outfield. He was then allowed to return
to pitching. He started with 10 to 15 pitches and gradually increasing the number of pitches each
session. Pitching mechanics were modified to improve recruitment of his trunk and hip extensors.
Overhead rather than side arm throws were encouraged. He was returned to game conditions 8 weeks
after the diagnosis.

- 168 -
IOC Sport Medicine Manual 2000
C. Wrist and Hand

1. Distal Radial Fracture

Case History - Distal Radial Fracture

A 13-year-old gymnast was following her free exercise programme. She slipped on takeoff and
fell on an outstretched left arm. She was immediately aware of marked pain and deformity to her
left hand and wrist with inability to actively dorsiflex the wrist because of discomfort.

Examination revealed a deformity of the distal radius at the wrist joint with marked pain on
attempted motion of the area. Neurological examination showed normal radial, median and ulnar
nerve function. Radial artery pulse was present and normal.

Discussion
Differential diagnoses include:
a. acute wrist sprain
b. wrist fracture
c. carpal dislocation
d. epiphyseal injury

Initially, this injury was managed by splinting to protect the wrist and prevent
further injury. An ice pack was applied to minimize soft tissue swelling and relieve
pain. The gymnast was immediately referred to an emergency department with x-
ray facilities. The x-ray demonstrated a fracture of the distal radius with 15
angulation and dorsal displacement of the distal fragment. This is a common injury.
In young people this fracture normally occurs in the region of the epiphyseal plate.
Five different types of fractures have been described relating to epiphyseal injuries.
Each fracture type has certain risks associated with it, depending on the actual
anatomy of the fracture line. Where the epiphyseal plate has closed and the distal
radius has fractured, a Colles fracture has occurred. Not infrequently, the ulnar
styloid process is also fractured.

4 - Injuries to the Musculoskeletal


All intra-articular fractures require anatomical reduction to decrease the risks of late onset of
degenerative osteoarthritis in the involved joint. If the fracture does not involve the joint surface, then
a closed reduction aimed at reducing the deformity and restoring the anatomical alignment should be

System
performed.

Figure 4.17 Colles fracture.

- 169 -
IOC Sport Medicine Manual 2000
Once satisfactory reduction is obtained, the patient will require a period of casting that will vary from
3-6 weeks, depending on the age of the patient. In a person with open epiphyseal plates, a shorter
period of casting is possible. If the fracture is unstable, it may require the use of a cast that extends
above the elbow to reduce pronation and supination in the forearm that may contribute to the instability.

When managing wrist fractures or dislocations where the nerves and vessels may be compromised,
one must check for distal sensory change and circulation to the limb with particular emphasis on the
median nerve.

It is extremely important at the time of diagnosis to rule out the presence of a carpal disruption. In
particular, the relationship between the scaphoid and the lunate and other carpal bones should be
examined to ensure that the patient has not suffered a perilunar dislocation. Such a condition is often
missed and lateral x-ray views are necessary for the diagnosis. If such a dislocation occurs, early
reduction is necessary. Operative intervention is required for anatomic reduction, internal fixation
and ligament repairs if possible. The wrist is then immobilized between 6 and 10 weeks before intensive
physiotherapy is begun. When managing wrist fractures or dislocations where the nerves and blood
vessels may be compromised, distal pulses and sensation should also be examined.

This gymnast was diagnosed with a fractured distal radius and was treated with closed reduction
under anaesthetic. Once satisfactory alignment was obtained, she was placed in a short arm cast for a
period of six weeks. In a younger person with open epiphyseal plates, a shorter period of casting is
possible. Range of motion exercises for the digits, thumb and elbow were performed during the
immobilization period and she continued to work out on beam and floor exercises (in activity not
using her wrist and arm) to maintain her general conditioning and lower body strength. Once the cast
was removed, she started physiotherapy that consisted of active range of motion and isometric exercises
of the wrist and elbow. Modalities were also used to control swelling which included contrast baths,
interferential current and ice. Strengthening was progressed to include concentric wrist, finger and
elbow exercises with tubing and finally eccentric exercise (Figure 4.18) was added. Grip strengthening
included squeezing water from a sponge and was progressed to squeezing a rubber ball. Proprioception
retraining was started at 6 weeks post-injury and included exercises such as wall push ups, prone
lying on the physiotherapy ball and weight-bearing through the upper limbs on the wobble board. She
4 - Injuries to the Musculoskeletal

was encouraged to return gradually to more sport specific activity such as chin-ups and handstands
once she could weight-bear fully through the upper limbs pain-free. She was allowed to return to the
uneven bars at 9 weeks and to full training at 12 weeks post-injury.
System

Figure 4.18 Eccentric wrist extension exercise - slowly lower wrist.

- 170 -
IOC Sport Medicine Manual 2000
2. Sprained Wrist

Case History - Sprained Wrist

A 26-year-old cyclist fell on his outstretched right hand when he was involved in a collision
with another cyclist. Fortunately, he sustained no major injuries but was aware of immediate pain
in his wrist area and tenderness was over the scapholunate region dorsally.

There was no history of previous injury to the thumb. Clinical examination revealed a decreased
range of motion at the wrist with tenderness in the anatomical snuff box and pain on attempted
motion of the thumb. Neurovascular function was normal, but swelling was present over the distal
radial area of the wrist. The metacarpal joints were not swollen or tender and the
metacarpophalangeal joint of the thumb was normal.

Discussion
Without the benefit of plain x-ray, the differential diagnosis would include:
a. sprain of the wrist (scapholunate ligament tear)
b. Bennetts fracture at the base of the first metacarpal
c. scaphoid fracture
d. metacarpalphalangeal joint ulnar collateral tear (skiers thumb)

X-ray examination is vital to further delineate the extent of the injury and separate bony from soft
tissue injury. This should include postero-anterior and lateral wrist views with specific x-ray of the
scaphoid, as scaphoid fractures are often difficult to see on normal views of the wrist. In the present
case, an undisplaced linear fracture was identified through the waist of the scaphoid. Intercarpal angles
(radolunate, scapholunate) were normal.

A Bennetts fracture is an intra-articular fracture dislocation at the base of the first metacarpal. More
often than not, this results in lateral displacement of the shaft of the first metacarpal in relation to the
medial aspect of the thumb. Should this occur, referral to a centre with the ability for operative reduction
of the fracture is appropriate. Failure to gain adequate anatomical reduction of this intra-articular
fracture will result in degenerative arthritis of the joint.

4 - Injuries to the Musculoskeletal


If the ligaments in relation to the metacarpophalangeal joint of the thumb have been damaged (ie.
skiers thumb), then adequate protection in a cast is recommended. Casting is required for a period of
6 weeks followed by protective taping with active mobilization. If there has been avulsion of a small

System
portion of bone or complete disruption of the ligaments with no end-point, operative repair is often
indicated, as this condition can result in significant weakness in grip strength in later years. Many of
these injuries require surgical exploration to anatomically restore ligamentous rupture and allow normal
recovery of function.

If a scaphoid fracture is identified, usually no reduction is required. A period of prolonged


immobilization is necessary, however, averaging from 10-15 weeks to allow adequate healing. The
reason for the lengthy period of immobilization is because past experience has demonstrated problems
with delayed healing presumably related to inadequate blood supply to the scaphoid. The scaphoid
receives its blood supply mainly through the dorsal aspect of the distal pole. When a fracture has
occurred through the waist of the scaphoid, the blood supply may become disrupted and complications
such as non-union and avascular necrosis can occur. The scaphoid cast should hold the wrist in
approximately 20 of dorsiflexion, include the thumb to its interphalangeal joint, include the wrist
and hand to the MCP joints and extend two-thirds of the way up the forearm.

- 171 -
IOC Sport Medicine Manual 2000
After 15 weeks of immobilization, if there is not satisfactory union or if avascular necrosis has occurred
at the proximal pole, then a further period of immobilization may be undertaken or, if necessary,
surgical intervention undertaken with open reduction and bone grafting. Following immobilization,
physiotherapy with range of motion exercises and strength training should be provided. An elite athlete
is often managed by immediate internal screw fixation permitting an earlier return, depending on the
sport.

Should a scaphoid non-union occur and not be recognized, abnormal motion between the proximal
and distal rows of the carpal bones may occur, leading to degenerative osteoarthritis (particularly in
the radial aspect of the wrist) and long-term limitation of wrist motion and loss of function.

If the x-ray is not diagnostic but history and examination produces suspicion, the wrist should be
immobilized in a cast for a period of two weeks. Following this, the cast should be removed and a
repeat x-ray of the wrist performed. If at this point there is no fracture apparent on plain x-ray, it is
usually safe to treat this as a wrist sprain and not recast. If the pain persists however, casting is
recommended and a bone scan is used to screen for a fracture.

For the wrist sprain that does not improve within 2 to 3 months, injury to the scapholunate ligament or
triangular fibrocartilage should be suspected. Wrist arthroscopy may be indicated to assess these
structures. Repair or debridement of the triangular fibrocartilage complex can be performed during
wrist arthroscopy. Depending on the degree of scapholunate instability and presence or absence of
arthritis, ligament reconstruction or limited intercarpal arthrodesis may be necessary for scapholunate
ligament tears.

The cyclist was diagnosed with a scapholunate ligament sprain with pain in the scapholunate region
and negative x-ray for a scaphoid fracture, and was immobilized in a splint for 2 weeks. Initial
management consisted of anti-inflammatories, active ROM exercises of the digits and ice was applied
to the wrist 4 times per day for 10 to 15 minutes. He was able to continue with a light cycling programme
with use of a tri-bar to prevent any weight-bearing through the wrist. Active ROM exercises of the
wrist and thumb were begun 2 weeks post-injury. Modalities were used to control swelling which
included pulsed low dose ultrasound, interferential current and laser. Strengthening exercises were
started at 2 weeks post-injury with isometrics and were progressed to include concentric and eccentric
4 - Injuries to the Musculoskeletal

exercise of wrist (Figure 4.18) and thumb (Figure 4.19). Proprioception retraining was started at 6
weeks post-injury and included wall push ups, upper limb weight-bearing prone on the wobble board
and the physiotherapy ball. The athlete was able to return to full training at 6 weeks post-injury with
the wrist taped and was able to discontinue use of the tri-bar on the bicycle.
System

Figure 4.19 Eccentric extension and abduction exercises of the thumb - slowly lower weight.

- 172 -
IOC Sport Medicine Manual 2000
3. Mallet Finger

Case History - Mallet Finger

A 20-year-old basketball player was struck on the tip of his right middle finger by a ball while
attempting to recover a pass. He had immediate pain and disability, and the pain persisted into the
next day.

Discussion
Differential diagnoses include:
a. fracture to the distal phalanx with or without a subungual haematoma
b. dislocation of the distal interphalangeal joint
c. avulsion of the flexor digitorum profundus
d. injury to the extensor tendons of the distal phalanx

Figure 4.20 Mallet finger.

Diagnosis will be made by physical examination and confirmed by x-ray.

A fracture to the distal phalanx with a subungual haematoma occurs most frequently from a direct,
crushing injury to the distal phalanx. On examination, there will be swelling and tenderness over the

4 - Injuries to the Musculoskeletal


distal phalanx with an obvious haematoma or bluish reaction under the nail that will be extraordinarily
tender. X-ray will confirm a fracture; these usually occur in the tip or in the body of the phalanx.
Treatment should include drainage of the subungual haematoma and splinting for comfort of the
fractured phalanx.

System
A dislocation of the distal phalanx is obvious on inspection and can be confirmed by x-ray. The dorsal
dislocation is the most common, and is caused by a forced hyperextension to the tip of the finger.
Reduction and immobilization for three weeks is indicated.

Avulsion of the flexor digitorum profundus may be a more difficult diagnosis to make. On careful
inspection, there will be a loss of active flexion at the distal interphalangeal joint. The history will
reveal that the athlete, in gripping the jersey or sweater of an opponent, will have a forced extension of
the distal phalanx with subsequent active flexion. This break-away motion avulses a flake of the distal
phalanx on the flexor surface. It may be a rather small finding in the x-ray. There are three levels of
this injury where avulsion without retraction may be present. Conservative treatment by applying a
splint to the finger may be all that is necessary. If there is retraction of the tendon along the flexor
surface of the phalanges or into an area proximal to the palmar area, surgery to reattach the flexor
digitorum profundus is necessary.
- 173 -
IOC Sport Medicine Manual 2000
Examination of the 20-year-old basketball player showed a loss of active extension of the distal phalanx.
He was found to have approximately 20 of flexion with an inability to extend that phalanx. This
occurred as the ball struck the tip of the finger when it was in active extension, producing a forced
flexion. This led to injury to the extensor tendon of the distal phalanx. X-ray examination revealed a
small flake of avulsed bone from the base of the distal phalanx on the dorsal surface and accounted for
the lack of active extension of the distal phalanx.

Injuries to the extensor mechanism can occur in one of five ways:


1. stretch injury
2. rupture of the extensor tendon
3. avulsion fracture
4. true fracture to the full base of the distal phalanx
5. epiphyseal injury if growth plate not closed

The basketball player was splinted for 8 weeks in an aluminium splint that held the distal interphalangeal
joint in extension and the proximal interphalangeal joint in flexion. During the immobilization period,
he was allowed to remove the splint several times a day to clean the skin and prevent maceration. Care
was taken to maintain the distal phalanx in extension. He was also advised to rest from his sport and
anti-inflammatories were prescribed. He was allowed to remove the splint for physiotherapy sessions
which included active range of motion exercises of the proximal interphalangeal joint, metacarpal
phalangeal joint, thumb and other digits, isometric wrist and forearm exercises, followed by contrast
baths. He was allowed to continue cycling, running, line drills, jumping activities, and a lower body
weight programme to maintain his general fitness.

At 8 weeks post-injury, his active finger extension was restored and he was allowed to discontinue use
of the splint. A night splint was worn for an additional 4 weeks after the day splint was removed.
Isometric exercises of the DIP joint and grip strengthening exercises were started at 8 weeks post-
injury followed by concentric and eccentric exercises with an elastic band 2 to 3 weeks later.
Proprioceptive retraining included sport specific drills such as ball handling, dribbling, shooting and
passing with the injured digit buddy taped to the other fingers. This basketball player was returned
to sport at 10 weeks post-injury and the injured digit was buddy taped for an additional 2 months. If
the athlete patient was an elite basketballer, he may play during the time required to heal this injury
4 - Injuries to the Musculoskeletal

wearing his mallet finger splint.

4. Acute Tenosynovitis of Extensor Tendons


System

Case History - Acute Tenosynovitis of Extensor Tendons

A 19-year-old rower presents with a three-day history of increasing pain in the back of her left
wrist. It had come on during distance training. She complained of marked pain and discomfort as
she attempted to flex and extend her wrist during the recovery phase of her stroke. Following
training, she had noted a grating sensation on the posterior aspect of the wrist. There was some
tenderness on direct palpation over the wrist area and soft tissue crepitus was noted. There had
been no other traumatic injury to the area apart from an episode that occurred one week earlier as
she struck the back of her hand against a wall.

Discussion
Differential diagnoses include:
a. acute tenosynovitis to the extensor tendons on the back of the wrist
b. bony injury from the direct trauma

- 174 -
IOC Sport Medicine Manual 2000
Bony injury would be excluded with an x-ray of the area.

Acute tenosynovitis is treated initially with a resting splint, the use of anti-inflammatory medication,
and ice massage every four hours to reduce the inflammation. In addition, the patient should be instructed
in a rehabilitation programme to regain flexibility and strength of her wrist and instructed in a gradual,
safe return to her sport.

Should the condition not settle in 7-10 days, consideration may be given to injecting the affected
tendon sheath with a corticosteroid preparation. Occasionally, surgical tenolysis and decompression
is required for chronic tenosynovitis.

This injury is also seen with swimmers using hand paddles and with canoeists.

This rower was diagnosed with an acute tenosynovitis and was treated initially with rest, ice massage
every 4 hours, anti-inflammatories and she was immobilized for 1 week in a resting splint. Physiotherapy
included pulsed low dose ultrasound, interferential therapy and ROM exercises of wrist and fingers.
Exercises were progressed to include shoulder and scapular exercises with surgical tubing (Figure
4.21), isometric exercises of the wrist extensors and a progression of deep abdominal exercises to
maintain trunk stability. Training was resumed at 2 weeks post-injury on the ergometer with a twice-
daily technical row of 400 m. Her symptoms were monitored over the next 48 hours and the intensity
and distance of her workout was increased by 400 m every 3rd training session as long as the pain did
not increase. She resumed training on the water at 3 weeks post-injury with her wrist taped until the
full intensity and distance of her regular work-outs were achieved.

4 - Injuries to the Musculoskeletal


System
Figure 4.21 a) Sweep and b) scull seated rows with surgical tubing.

- 175 -
IOC Sport Medicine Manual 2000
D. Spine

1. Lumbosacral Disc Disease

Case History - Lumbosacral Disc Disease

A 26-year-old rower presented with a four-week history of increasing right buttock pain. The
pain initially occurred while doing heavy squats in weight training, and as the athlete attempted
to train through the pain, the pain persisted. With ongoing activity in rowing and weight training,
the pain tended to radiate down the back of his right leg and, on occasion, would reach to his right
heel. It was worse while sitting. He gave no history of any sudden alteration of his training regime
and no history of major injury. He related the onset to an episode of weight-training when he was
performing a squat with a free weight, lost his balance, twisted to the side, and felt a sudden
twinge of pain in his back. There was some local discomfort of the lumbosacral junction of the
spine.

On physical examination, he was able to stand erect and had decreased mobility and pain on
forward flexion of the spine. He had some slight weakness of the calf muscles in his right leg and
a decreased ankle jerk on the right side as compared to the left. There was some minor sensory
disturbance along the lateral border of his right foot when tested with pin-prick sensation. Straight
leg raising was limited on the right side to 60 compared to 90 on the left. The rest of his neurological
examination was within normal limits.

Discussion
Differential diagnoses include:
a. disc prolapse and nerve root irritation
b. spondylolysis
c. pyriformis syndrome
d. hamstring syndrome
e. referred pain from visceral causes
f. vascular pain from peripheral insufficiency

The pain pattern with disc prolapse and nerve root irritation is often of recent onset related to a particular
4 - Injuries to the Musculoskeletal

movement when the low back is flexed and rotated; eg. in lifting, or in hitting the backhand in tennis.
The athlete may have had prior episodes that involved the low back with or without leg involvement.
If lower extremity muscular or sensory involvement is noted, the likelihood of disc prolapse is very
high. The pain is worse with sitting, sleeping face down, bending forward, and in daily activities such
System

as brushing teeth. Often there is a progression of lower extremity pain over time. Initially, pain may
radiate to the buttock, then progress to the posterior thigh, the posterior calf, and to the sole of the foot.
There may or may not be concomitant numbness
and weakness. Physical findings may include
reduced and painful lumbar flexion, reduced
straight leg raising and neurologic signs, as noted
in this rower. The distribution of pain and sensory
changes may indicate the level of the disc prolapse
(see figure 4.22).

Spondylolysis is a break in the pars intra-


articularis of the vertebra. Pain is experienced
predominantly on extension of the spine, rather
than on flexion, and neurological signs are

- 176 -
IOC Sport Medicine Manual 2000
uncommon. Lateral and oblique x-ray views should be diagnostic (see below for further information
on spondylolysis).

An uncommon cause of buttock and leg pain is the piriformis syndrome. This involves spasm of the
piriformis muscle that compresses the sciatic nerve that exits from the pelvis between or below the
muscle fibres. The cause of the muscle spasm is often direct trauma although it can come on in a
gradual fashion. The pain pattern is identical to the disc prolapse but without low back pain. There can
be concomitant numbness, tingling and weakness in the leg. Physical examination would include an
area of tenderness and possible spasm in the mid-buttock over the piriformis muscle with a positive
straight leg raise sign and neurological signs.

The hamstring syndrome is similar to the piriformis syndrome and is also uncommon. It involves
trapping of the sciatic nerve by a fibrous band that forms part of the origin of the hamstring muscles
near the ischial tuberosity. The pain pattern is similar to the piriformis syndrome but with proximal
posterior thigh pain at the hamstring origin instead of the buttock pain. There is a similar pattern with
respect to the distal pain - numbness, tingling and weakness. Often the pain is worse with hamstring
stretching, sitting and sprinting. Physical findings include point tenderness over the hamstring origin
with a positive straight leg raise test and possible neurologic signs. Often, conservative treatment is
not helpful but surgery is successful, allowing full function in over 80% of the cases.

While viscerogenic pain may radiate to the back, particularly in pancreatic and renal disease, it is rare
to have neurological findings in the lower extremity. Low back pain in females that is increased
premenstrually may be due to endometriosis, pelvic pathology or metastatic disease and these causes
must be excluded. Vascular causes of pain in the lower extremity, while reasonably common in older
people, would be rare in fit, relatively young athletes.

In our rower, plain x-ray of the lumbar spine showed some minor disc space narrowing at the L5-S1
level but no bony abnormality. A CT scan revealed a rightsided bulging disc at the L5-S1 level with
minor impingement of the S1 nerve root and excluded any major disc prolapse with herniation.

4 - Injuries to the Musculoskeletal


System

Figure 4.22 Radiation of pain from L4, L5, S1.

- 177 -
IOC Sport Medicine Manual 2000
Initial treatment is non-operative, and involves a period of rest and the use of anti-inflammatories to
reduce the local inflammation and the sciatic nerve irritation. The athlete should be educated regarding
the avoidance of prolonged sitting, forward bending, lifting, or sneezing/coughing in a bent forward
position. In the initial stages, local modalities such as ice, ultrasound, or interferential current may be
useful. Of particular benefit is the use of manual or, if available, mechanical traction.

As soon as possible, a series of exercises aimed at recruiting the stabilizing abdominal muscles and
regaining range of motion should be initiated. Exercises can be graded to the stage of healing and
should remain pain-free. Abdominal supporting exercises (see Figure 4.23) should be initiated
immediately, as well as extension exercises in supine and standing (see Figure 4.24). Graded flexion
exercises can also be initiated if they remain pain-free, as well as neural mobilization exercises (Figure
4.25). A graduated return to sport can begin with swimming, water running, and seated-upright rowing
or cycling as tolerated.

With disc disease and nerve root irritation, if there is no recovery for a period of 4-6 weeks, options
such as chymopapain injection or epidural corticosteroid injection may be tried prior to surgical
intervention.

Non-operative treatment was successful in our 26-year-old rower. Two weeks of rest allowed the pain
to settle, after which he began a swimming programme and strengthening exercises aimed particularly
at his abdominal and paraspinal muscles. As his straight leg raising returned to normal (difference
between legs being < 5), he was allowed to train on the rowing ergometer. He returned to full training
at 8 weeks post-injury.
4 - Injuries to the Musculoskeletal

Figure 4.23 Abdominal supporting exercises - monitor neutral torso position with hands as legs flex
System

and extend.

- 178 -
IOC Sport Medicine Manual 2000
Figure 4.24 Extension exercises.

Figure 4.25 Neural mobilisation exercises.

2. Spondylolysis and Spondylolisthesis

Case History - Spondylolysis and Spondylolisthesis

4 - Injuries to the Musculoskeletal


A 22-year-old university high jumper presented with a history of increasing low back pain that
began in the lumbosacral region and radiated into both buttocks and upper thighs. He could not
relate the onset of his pain to any one specific incident and while he was still able to train with
discomfort, this pain intensified with increased activity and reduced with rest. He had no previous

System
history of back pain or spinal injury, although he had noticed occasional backache in the past.

Physical examination revealed no major abnormalities apart from a slight increase in the
normal lumbar lordosis. He had some minor discomfort on forced extension and rotation of the
lower lumbar spine. There was also a slightly decreased range of forward flexion. Lateral flexion
and rotation were normal. Neurological examination of his lower extremities including all major
dermatomes, peripheral nerve groups, and reflexes showed normal responses.

Discussion
Differential diagnoses include:
a. acute spondylolysis
b. chronic or congenital spondylolysis
c. mechanical low back pain
d. facet syndrome

- 179 -
IOC Sport Medicine Manual 2000
X-ray of the lumbosacral region, including oblique views, showed normal vertebral spaces. The major
finding was that of bilateral defects in the pars interarticularis of L5 (Figure 4.26). There was no
sclerosis of the facet joints and no forward slip of L5 on S1. Commonly, x-ray does not adequately
image the pars interarticularis area so a bone scan with SPECT should be ordered. A CT scan or MRI
would also be able to detect the lesion.

The major differentiation between acute and chronic spondylolysis is based on radiographic findings,
where the margins around the spondylolytic lesion will tend to be somewhat sharper in an acute
spondylolysis. A radionuclide scan (bone scan) or CT scan may also be helpful, if available, in
differentiating between acute and chronic lesions. Commonly, there will be increased uptake of nuclide
with spondylolysis. A CT scan will clearly define an acute fracture.

When a spondylolysis has been diagnosed, it is difficult to differentiate between acute and chronic
status. The initial treatment is conservative and is aimed at reducing pain and promoting healing by
using analgesics and modalities such as heat, ice, ultrasound, or interferential current. With acute
spondylolysis, a Boston anti-lordotic brace may be used to minimize motion, thus allowing the pain
to settle and healing to occur. A brace may be used for 6-8 weeks in conjunction with abdominal
stablisation and paraspinal strengthening exercises. The athlete should be educated regarding the
avoidance of forceful extension of the lumbosacral region for approximately 15 weeks. Pain-free
swimming, water running, cycling, and light running at the later stages would be appropriate activities
to maintain fitness.

For chronic spondylolysis with an acute flare-up of an established non-union, treatment is aimed at
reducing the pain and recruiting the abdominal muscles for support. Anti-inflammatories may be
needed. The use of a lumbosacral brace is not recommended and return to activity is allowed at an
earlier stage.

When performing muscle-strengthening activity, particular attention should be paid to increasing the
flexibility of the hamstrings. One of the complicating factors of a spondylolysis is marked decrease in
hamstring flexibility with hamstring spasm.
4 - Injuries to the Musculoskeletal

Spondylolysis can occur in 3-4% of the general population, but these numbers can be 3-5 times higher
in the athletic population (eg. gymnastics, diving, high jumping) where there is an increased
hyperextension and rotation which stresses the posterior elements. The condition may be completely
asymptomatic. If spondylolysis occurs acutely as a result of stress in activities that are not modified,
System

it may progress to a spondylolisthesis, where the body of the proximal vertebra slides forward in
relation to the lower vertebra. Should this happen acutely, the spondylolytic crisis has occurred and
this requires immediate withdrawal from all strenuous activities.

- 180 -
IOC Sport Medicine Manual 2000
Figure 4.26 Spondylolysis (left) showing a defective vertebral arch. Spondylolisthesis (right) where
there is slippage of the vertebra forward in relation to the one below.

Spondylolisthesis can be graded depending on the degree of forward slip of the superior vertebral
body in relation to the inferior body. In mild cases where less than 30% of forward slip is present,
treatment is undertaken in the same manner as that for an acute spondylolysis. However, if, after an
adequate period of approximately 4-6 months of rehabilitation, resumption of activity shows a return
of symptoms and a continued feeling of instability of the spine, then referral to a surgical centre for
possible surgical stabilisation of the lower spine would be indicated. Often the spondylolisthesis is
accompanied by symptoms of nerve root irritation as the nerves exiting the spinal foramina in the
region of the pars interarticularis are irritated and compromised by the area of non-union.

Spondylolysthesis treatment is undertaken in the same manner as that for an acute spondylolysis.
Therapeutic modalities as above may be used initially to decrease pain and promote healing. Exercise

4 - Injuries to the Musculoskeletal


should again include abdominal stabilisation exercises in combination with the use of posterior pelvic
tilts to reduce the forward slippage during these exercises (See Figure 4.27). Forward flexion exercises
may also be useful to encourage normal range of motion and decrease pain. The athlete should be
educated regarding the avoidance of activities that may encourage further slippage of the disc such as

System
prolonged walking, downhill walking or running, and sitting. If these activities must be undertaken,
incorporating a posterior pelvic tilt will help to decrease the amount of forward slippage. In returning
to activity, the athlete may begin with pain-free swimming (avoid dolphin kick), cycling with a pelvic
tilt, and light running with a pelvic tilt.

- 181 -
IOC Sport Medicine Manual 2000
Figure 4.27 Posterior pelvic tilts - maintain tilt as opposite limbs flex and extend.

The high jumper was diagnosed as having a chronic spondylolysis, which had previously been
asymptomatic. The diagnosis was aided by reviewing a previous set of x-rays of his lower lumbar
spine that had been taken for an unrelated problem. He ceased all physical activity, and was given
anti-inflammatories and analgesics. His pain settled and he became asymptomatic. He was placed on
a rehabilitation programme aimed at strengthening the abdominal and paraspinal muscles. He was
allowed to return to weight training at approximately eight weeks following the onset of pain. He
returned to running at the same time with instructions to avoid downhill running. He began high
jumping at 75% effort at 12 weeks and full effort at 15 weeks. He returned to full, active competition
at 20 weeks post-injury.

3. Facet Joint Syndrome

Case History - Facet Joint Syndrome


4 - Injuries to the Musculoskeletal

A 25-year-old long distance runner presented with a history of increasing low back pain and
discomfort over several weeks following an increase in training mileage and hill training. He was
aware of the pain while running downhill but not as he ran uphill. Running on flat land produced
some back pain but at a lower intensity. The pain did not radiate to the buttocks or legs.
System

Physical examination revealed a normally aligned spine with a full range of forward flexion.
Extension of the lumbar spine was limited by pain. Lateral flexion and rotation were within normal
limits. Neurological examination of the lower extremities (including touch and pain sensation,
deep tendon reflexes, muscle strength and proprioception) was within normal limits.

Discussion
Differential diagnoses include:
a. mechanical low back pain with facet joint syndrome
b. a bulging intervertebral disc
c. spinal stenosis
d. spondylolysis
e. non-musculoskeletal causes (rare)

- 182 -
IOC Sport Medicine Manual 2000
Facet joint syndrome is due to repeated hyperextension of the low spine without necessarily a discrete
injury, and is commonly seen in gymnastics, weightlifting and football. The action of hyperextension
produces compression of the facet joints and increases synovial irritation around these joints. It presents
with aching low back pain, exacerbated by running downhill and is eased somewhat by the forward
flexion of running uphill. Major treatment of this problem involves modification of the training
programme with the use of anti-inflammatories to reduce the acute inflammation and modified activity
to reduce loading on the facet joints.

Spinal stenosis often presents with a history of pain that comes on with activity and tends to ease
with rest. The pain tends to be symmetrical in its distribution and often mimics vascular claudication
type pain. It is not usually confined to any specific area but rather produces a generalized feeling of
discomfort and heaviness in the lower lumbar region. In addition, the pain is precipitated, aggravated,
and progressively intensified by activity, and eased by rest. It differs, however, from claudicant pain
and peripheral vascular insufficiency in one important aspect; the pain is experienced first in the
proximal rather than the distal limb. The diagnosis is confirmed by a CT scan or MRI that will
determine the degree of stenosis present.

Intervertebral disc herniation is a frequent cause of low back pain even in the absence of physical
findings such as a positive straight leg raising test, depressed deep tendon reflexes, hyperaesthesia or
motor deficits. If x-ray examination is normal and the history is suggestive, a CT scan or MRI may
confirm a disc bulge or herniation. However, in these situations, a conservative approach including
activity modification and rehabilitative exercises is very often successful in bringing about resolution
of the symptoms.

Other causes of back pain should be considered if symptoms are not responsive to treatment and
clinical findings have not changed.

X-ray examination of the lumbar spine of our long distance runner showed no bony abnormalities
and, in particular, no disc space narrowing, no evidence of Scheuermanns disease and no evidence of
facet joint degeneration. He was diagnosed as suffering from facet joint syndrome because of the
characteristic history of the pain and the reproduction of symptoms on back extension.

4 - Injuries to the Musculoskeletal


Treatment of this problem involves the use of anti-inflammatories to reduce acute inflammation as
well as local therapeutic modalities such as ice, heat (sub-acutely), ultrasound, and interferential
current. The athlete should be educated regarding the avoidance of painful positions such as

System
hyperextension during standing or walking, or prolonged/extreme positioning in flexion or extension
such as running up or down hills. Exercises should include abdominal supporting exercises, flexion
and rotation range of motion exercises as well as pain-free self mobilization extension range of
motion exercises. Manual traction, or if available, mechanical traction, may be useful to decrease
pain.

Fitness may be maintained through cycling and water running. As the athlete begins to regain more
pain-free range of motion, light running may be initiated as well as sport specific training.

The long distance runner reduced his training volume by 50% and restricted himself to flat land
training for a period of four weeks until the acute pain settled. Over the next few weeks he increased
his training to its previous level but minimized running down long and steep hills.

- 183 -
IOC Sport Medicine Manual 2000
4. Cervical Nerve Root Injury versus Brachial Plexus Injury

Case History - Cervical Nerve Root Injury

A 24-year-old wrestler attempted to shoot for his opponents leg during a match. He was blocked
by the opposing player and suffered an injury to the head and shoulder area. There was a sudden
sharp burning pain in the trapezius area that radiated down into the hand. This severe pain lasted
approximately one minute, however, the ache down into the hand continued for another couple of
minutes with an associated feeling of weakness in the affected arm. At his clinical assessment, the
wrestler had loss of extension of the neck due to discomfort and a positive Sperling manoeuvre to
the affected side (see Figure 4.28). Neurological examination revealed normal sensation to light
touch and pinprick. There was significant loss of elbow flexion strength on the affected side. Lower
extremity examination was normal.

Figure 4.28 Positive Sperling manoeuvre - reproduction of nerve conductivity signs and symptoms
with full neck extension, side flexion, and rotation.

Discussion
Differential diagnoses include:
a. acute cervical spine dislocation or fracture
b. acute nerve root injury secondary to either cervical disc disease or cervical stenosis
4 - Injuries to the Musculoskeletal

c. acute muscle strain


d. brachial plexus traction injury

Recognition of a cervical spine dislocation and/or a fracture is critical to prevent catastrophic injury in
System

sport. Quadriplegia and even death can occur from injuries to this area. It is imperative to prevent any
extension of this injury.

In these situations, it is extremely difficult to be absolutely sure of the mechanism of injury. These
injuries occur most commonly in sports where there is a collision at high speed between opponents,
and in mechanized activities such as cycling and water skiing. Bilateral upper extremity and/or lower
extremity symptoms or physical findings, such as bilateral weakness or sensory deficits, should alert
the attending health care worker to significant cervical spine pathology.

The injury presented by this wrestler is indicative of cervical nerve root injury. In this case, the C6
nerve root has been affected. Most commonly this is related to some mild stenosis of the C5-6
neuroforamina by early degenerative disc disease. The attending medical practitioner should carefully
assess motor and sensory function, as sometimes the findings are subtle. It is imperative with neurologic
function loss and decreased range of motion of the cervical spine that the athlete is removed from the

- 184 -
IOC Sport Medicine Manual 2000
competition and completely and carefully assessed. Risk of catastrophic injury and possible permanent
nerve root injury, can be prevented by following this strategy.

Brachial plexus injury has been considered to be more common than cervical nerve root injury but
thoughts on this have changed in recent years. The brachial plexus injury is a stretch of the brachial
plexus that is transient in nature. Usually by the time the attending medical practitioner evaluates the
patient, the pain, as well as the numbness and tingling into the shoulder and arm, have completely
resolved. There should be no loss of cervical range of motion and no neurological findings of the
upper extremities with brachial plexus injury. If an athlete presented with this scenario and physical
examination revealed full cervical range of motion and no neurologic dysfunction, return to play can
be permitted immediately.

The athlete with decreased cervical range of motion and neurological dysfunction needs to be properly
assessed with x-ray. If there has been multiple episodes of this type of injury, further assessment with
a computed tomography scan (CT) or magnetic resonance imaging (MRI) would be important to
delineate the cervical pathology.

Treatment of cervical spine nerve root injury involves educating the athlete to avoid activities that put
the cervical spine at further risk of injury, the use of anti-inflammatories, and local modalities to
decrease muscle spasm. Modalities may include ice, heat (sub-acutely), and ultrasound. Manual traction
or mechanical, if available, would be very useful in decreasing pain and pressure on the nerve root.
The athlete should be advised regarding avoidance of nerve tensioning positions (Figure 4.29), and
maintaining a proper upright cervical posture. Pain-free exercises can begin immediately and should
include cervical neck supporting exercises (Figure 4.30), cervical range of motion exercises, shoulder
range of motion exercises, and gradual nerve mobility exercises (see Figure 4.31).

4 - Injuries to the Musculoskeletal


System
Figure 4.29 Avoiding nerve tensioning positions for cervical spine - here showing median nerve bias
that might occur in weight training.

Figure 4.30 Cervical spine exercises - maintain neck position as therapist gradually removes supporting
hands.
- 185 -
IOC Sport Medicine Manual 2000
Figure 4.31 Gradual nerve mobility exercises - maintain cervical, thorax, and shoulder position with
either wrist extension and/or elbow extension.

Physical fitness may be maintained through the use of an upper body cycle ergometer bicycle, and/or
stairclimber/stepping machine; particular attention should be taken to ensure good cervical posture
during these activities. The athlete may gradually return to sport as pain-free cervical motion and
shoulder range of motion increase and positions of nerve tension can be obtained without symptoms.

The presence of a Horners syndrome would indicate a lower root injury at the C8-T1 level. Disturbance
in diaphragmatic function would indicate an upper cervical injury involving the phrenic nerve (C3-
C5).

Our wrestler withdrew from active competition at the time of his injury. He was then treated with anti-
inflammatories to reduce acute local inflammation. He was placed on a series of neck strengthening
and rehabilitation exercises. Specifically, he did stabilisation exercises combined with three sets of 20
manually resisted movements, which involved extension, flexion, side flexion, and rotation. Each
isometric movement was held for 10 seconds. These exercises were done under the supervision of a
physiotherapist. He did specific strength training of elbow flexion. He was able to return to full, active
competition once he obtained full neck range of motion, elbow flexion strength, and nerve tensioning
positions without pain.

E. Hip, Pelvis and Thigh


4 - Injuries to the Musculoskeletal

1. Femoral Neck Stress Fracture

Case History - Femoral Neck Stress Fracture


System

A 25-year-old female distance runner presents with a four-week history of right hip pain. The
pain spread anteriorly and medially in a diffuse pattern and became worse with running so that
she limped for several days after a run. Initially, the pain occurred only after running, then after
the first 10 minutes of running, and ultimately during the entire run until she had to stop. The onset
of pain was gradual and was relieved by rest. There was night discomfort described as a deep
ache. She was a regular runner until one year ago when she began cycling. Two months ago she
resumed running for 45 minutes per day five times per week, which was rapidly increased to one
hour a day at the end of the third week. The pain commenced during the fourth week of running.
There was no history of prior hip, back or leg problems. She was eumenorrheic.

She saw her family physician. X-ray imaging was interpreted as normal. He prescribed rest.
After two more weeks, she was walking pain-free and went out for a run. After 10 minutes ,she
experienced a flare-up of pain that took more than two weeks to settle.

- 186 -
IOC Sport Medicine Manual 2000
Physical examination revealed an inability to hop on the injured leg due to pain. Clinical
examination of her back and knees were normal. The hip range of motion was full with slight pain
at the extreme of internal rotation. Resisted hip flexion, abduction and internal rotation were
painful and weak compared to the uninjured side. Neurovascular examination was normal. Leg
lengths were equal.

Discussion
Differential diagnoses of hip pain are complex, and include:

Non-mechanical causes:
a. rheumatic disease
b. infectious process - including septic arthritis or proximal femoral osteomyelitis
c. neoplastic disease - including benign or malignant conditions
d. kidney - renal colic
e. genitourinary disease

Mechanical causes
i. soft tissue injuries
f. lower abdominal muscle strain
g. iliopsoas strain
h. rectus femoris strain
i. gluteus medius strain
j. trochanteric bursitis
k. adductor strain
l. lumbar strain/disc strain

ii. bony injuries


m. stress fracture - pelvis and/or proximal femur
n. osteitis pubis (pubic symphysitis)
o. acetabular dysplasia with osteoarthritis
p. spondylolysis with or without spondylolisthesis of the lumbar spine

4 - Injuries to the Musculoskeletal


Non-Mechanical Causes
Athletes are prone to the same non-mechanical problems as the general population. The
spondyloarthropathies including ankylosing spondylitis, Reiters syndrome and other non-infective,
inflammatory conditions can cause sacroiliitis and associated hip pain. It is often associated with

System
morning stiffness. The pain is often relieved with activity.

Septic arthritis and osteomyelitis present with severe pain during rest with marked exacerbation of the
pain with motion and rapid worsening of the pain over a short period of time. There may be
accompanying fever and chills.

In the case of low-grade osteomyelitis, a gallium radionuclide scan may be of use in attempting to
differentiate the persistent, chronic hip pain. An elevated C reactive protein is almost invariably present.
Early diagnosis is extremely important in the case of a septic arthritis; treatment with antibiotics and
possible open hip drainage is mandatory.

With benign or malignant neoplasms, the pain pattern is often diffuse, worse at night and at rest, and
eased with movement. The hip range of motion may or may not be affected.

- 187 -
IOC Sport Medicine Manual 2000
The presence of a neoplasm may require the use of CT scanning to demonstrate its presence. Once its
presence is located, biopsy is necessary for tissue diagnosis.

There are a number of intra-abdominal disorders, including those of a gynaecological nature that can
manifest themselves as hip pain. If the pain pattern does not fit for a musculoskeletal problem, then
assessment of the abdomen and internal pelvis should follow.

Mechanical Causes
a. Soft Tissue Injuries
The soft tissue injuries of the hip and pelvis that are common include overuse muscle strains of the hip
flexors or adductors and referred pain from the lumbar spine or sacroiliac region. The diagnosis is
made clinically by pain on passive stretching and active resistance of the involved muscles and
palpation.
4 - Injuries to the Musculoskeletal
System

Figure 4.32 Soft tissue injuries of the hip and pelvis.

Trochanteric bursitis and peritrochanteric insertional enthesitis may present with or without a history
of trauma where the pain is localized over the region of the greater trochanter and trochanteric bursal
area. The pain is exacerbated with hip motion while direct pressure is applied to the area.

Lumbar and sacroiliac problems can cause pain in the distant areas including the hip and pelvis.
Often, poor posture or a specific motion prompts the flare-up.

Treatment of soft tissue injuries includes reducing weight-bearing activity, or substituting with non-
weight-bearing activity such as swimming, deep-water pool running, cycling and rowing if pain-free.

- 188 -
IOC Sport Medicine Manual 2000
Ice, physiotherapy, non-steroidal anti-inflammatories, joint flexibility and muscle strengthening with
particular emphasis on strengthening the abdominal muscles are essential components for recovery.
Gradual reintroduction of activity completes this treatment.

b. Bony injuries
The bony injuries of the hip and pelvis can be differentiated based on physical examination. This is
supported by the use of plain x- rays and often a radionuclide scan (bone scan).

Stress fractures can occur at the femoral neck, proximal femur, and inferior and superior pubic ramus
with pain present in the proximal thigh, perineal region and the groin area. Pain is exacerbated by
activity and eased with rest. The hop test is usually painful. The range of motion of the hip joint will
be decreased when moved actively, but will be normal when moved passively. X-ray may not
demonstrate a major abnormality but increased uptake on the radionuclide scan is extremely useful in
making the diagnosis of osteitis pubis.

Osteitis pubis may present in a similar fashion to stress fractures of the hip and pelvis. The tenderness
is located over the symphysis pubis and there is a decreased hip range of motion, especially with
abduction. A radionuclide scan will confirm the diagnosis.

Avascular
Necrosis of
Spondylolysis,
Femoral
Spondylolisthesis
Head,
Slipped
Capitus
Femoris

Osteochondritis,
Osteoarthritis

4 - Injuries to the Musculoskeletal


Stress
Fractures Osteitis

System
Pubis

Figure 4.33 Bony injuries of the hip and pelvis.

Acetabular dysplasia is a condition that can present with hip pain in the young athlete. In the older
athlete this may have led to the onset of degenerative osteoarthritis in the hip, again with diffuse
anterior groin and lateral hip pain. Hip motion may be reduced, particularly rotation, abduction and
adduction. Previous childhood hip problems such as slipped capital femoral epiphysis or Legg-Perthes
disease may also present in later life with hip pain aggravated by activity.

Spondylolysis with or without spondylolisthesis of the lumbar spine may present with pain in the hip
area. The range of passive hip motion is full and non-painful, but there is often tenderness in the lower
back. There is also decreased range of motion in the lumbosacral spine region. There may be a step

- 189 -
IOC Sport Medicine Manual 2000
deformity noted or an increase in the normal lumbar lordosis. Oblique x-rays may often reveal a
spondylolysis and lateral x- rays best demonstrate the spondylolisthesis.

Avascular necrosis of the femoral head may also present with hip pain in its more advanced state.
There are multiple causes of avascular necrosis but it should be considered in the differential diagnosis
of a young athlete who has been on long-term corticosteroids or who has been involved in scuba
diving in the past. The diagnosis is confirmed by x-ray and is treated accordingly.

In the case of the 25-year-old distance runner, x-ray of the hip and pelvis were found to be normal. A
radionuclide scan revealed an abnormality in the right femoral neck. The diagnosis is a stress fracture
of the femoral neck, predominantly due to a training error. The need to make the correct diagnosis is
critical. Potentially devastating consequences can ensue from missing the diagnosis. The stress fracture
can progress to a complete fracture with displacement that disrupts the blood supply proximally,
resulting in avascular necrosis of the femoral head and degenerative osteoarthritis. The radionuclide
scan is the test to exclude or confirm this diagnosis.

The risk factors leading to hip and pelvis problems including femoral neck stress fractures are:
 leg length discrepancy
 weak abductors leading to positive Trendelenburgs sign and hip external rotators
 overuse injuries: too much exercise too soon; too hard, soft, hilly or uneven training surfaces;
poor strength and flexibility; poor shoes; poor biomechanics when running

Women appear to be at greater risk of stress fractures of the hip and pelvis due to:
 wider pelvis
 genu valgum
 less muscle strength
 less bone mass
 possible osteoporosis if low estrogen levels secondary to amenorrhea or hypoestrogenic state
The treatment of femoral neck stress fractures is aggressive and involves close clinical and x-ray
4 - Injuries to the Musculoskeletal

follow-up to ensure that the fracture is not progressing. This is done by clinical assessment and repeat
x-ray. At least one repeat x-ray is important at 4-6 weeks following the diagnosis to ensure that no
displacement of the stress fracture has occurred.
System

In this dangerous stress fracture, the athlete should be placed on non-weight-bearing crutch use until
they can walk pain-free with no end of day ache. Once the athlete is pain/ache free and there is evidence
of fracture healing, as shown on repeat x-ray and possible repeat radionuclide scan, gradual
reintroduction of activity is initiated. This begins with an alternate day programme starting with walking
and physiotherapy. Non-weight-bearing aerobic activities such as cycling, swimming and deep-water
running should be introduced to maintain aerobic fitness once the athlete resumes weight-bearing.
Initial physiotherapy treatment would include gentle stretching (hip flexor and outer hip stretches), as
well as a progressive strengthening programme for the core stability muscles. Initial hip strengthening
exercises should include hip rotation strengthening (Figure 4.34) and progress as tolerated to more
advanced gluteal strengthening (Figure 4.35) as well as proprioceptive and walk-run exercises.

- 190 -
IOC Sport Medicine Manual 2000
Figure 4.34 Hip rotation strengthening exercises - maintain neutral torso position
a) with externally rotated hip against resistance
b) with internally rotated hip against resistance.

Figure 4.35 Gluteal strengthening - with hips together, maintain neutral spine while abducting and
externally rotating the hip against resistance.

4 - Injuries to the Musculoskeletal


The 25-year-old runner with a femoral neck stress fracture responded to the treatment in the predictable
manner. She was pain-free in her daily activity within seven weeks. She commenced her rehabilitation
programme at 10 weeks and was running 15-30 minutes on alternate days at 14 weeks.

System

- 191 -
IOC Sport Medicine Manual 2000
2. Apophyseal Injury

Case History - Apophyseal Injury

At the beginning of the high school track and field season, a 14-year-old hurdler was doing his
last repeat of a workout when he suffered a sudden, painful snap in the lower buttock on extension
of his lead leg over the hurdle. The pain was acute and did not allow further training. He limped
home and placed ice on the painful area. The next day he could not walk without a significant limp
and sought medical help.

He had been perfectly healthy with no prior history of athletic injury other than the occasional
ankle sprain. Physical examination revealed a healthy adolescent with a limp, who couldnt hop,
do a full squat on the right side or flex and extend his back due to pain in the gluteal region. Hip
and back range of motion was reduced in all ranges, with flexion and extension being especially
painful. Resisted hip extension and knee flexion were impossible due to pain. Strength on resisted
hip flexion, abduction and adduction was reduced but present.

Discussion
The differential diagnoses of hip and buttock pain in a pre-adolescent or adolescent athlete with the
above findings should include:
a. trauma to the spine, pelvis, hip joint region or femur
b. infection of the hip joint or adjacent bone
c. referred pain from the abdomen

It is essential that an x-ray be performed and where there is skeletal immaturity, both sides of the body
need to be x-rayed for reference purposes.

An x-ray of his pelvis and hip revealed an avulsion of the ischial tuberosity at the origin of the right
hamstring (Figure 4.36).
4 - Injuries to the Musculoskeletal
System

Figure 4.36 X-ray showing avulsion of ischial tuberosity.

Prior to full bone maturation, the tendon-bone junction is notable by the presence of the apophysis.
The apophysis is a bony projection that is connected to the bone by a cartilaginous growing plate
(physis). It serves as an attachment point for tendons and as such is under tension. The cartilaginous
- 192 -
IOC Sport Medicine Manual 2000
plate allows the tendon to maintain its insertion position on the growing bone and allows the bone to
continue to grow in relation to the apophysis. It is, however, the weak link in the muscle-tendon-bone
chain.

The apophysis that is most frequently involved in clinical problems is that of the attachment of the
patellar tendon at the tibial tubercle region on the anterior surface of the proximal tibia. Osgood-
Schlatters disease is a chronic injury of this apophysis. Traction apophysitis may precede the more
dramatic avulsion injury.

Apophyseal injury can present as a traumatic or overuse injury. Traumatic injury, as in this case
presentation, is characterized by a sudden onset of severe pain during an athletic endeavour, whereas
the gradual onset of pain and discomfort at the region of the apophyseal plate signifies an overuse
type of injury.

a. Traumatic
There are five areas of the pelvic region (Figure 4.37) where the common traumatic apophyseal injuries
occur (in order of frequency):
1. anterior inferior iliac spine at attachment of rectus femoris
2. ischial tuberosity where the origin of the hamstrings are attached
3. anterior superior iliac spine at the attachment of the sartorius muscle
4. iliac crest at the attachment of the abdominal and superior part of gluteus medius muscle
5. lesser trochanter at the attachment of the iliopsoas tendon

Iliac
Crest

Anterior
Superior
Iliac Anterior

4 - Injuries to the Musculoskeletal


Spine Inferior
Iliac
spine

Lesser

System
Trochanter Ischial
tuberosity

Figure 4.37 Sites of hip/pelvis avulsion injuries.

The diagnosis of apophyseal injury is made by history, physical examination and appropriate x-ray
studies. Treatment depends on the degree of displacement at the apophyseal separation. If displacement
is wide (more than two cm), then surgery is indicated. Most commonly the treatment is non-operative.
Pain can be controlled and activity modified to allow the area to heal. Flexibility exercises are then
introduced along with strengthening exercises.

- 193 -
IOC Sport Medicine Manual 2000
A complication of an avulsed apophysis is the possibility of heterotopic ossification, particularly
around the major joints such as the hip, which can impair range of motion and function if severe
enough. If recognized, the initial treatment for a heterotopic ossification is rest to reduce the stimulus
to new bone formation followed by gradual mobilization activities.

b. Overuse
When an adolescent athlete presents with gradual onset of localized periarticular pain, it is important
to consider the presence of a traction apophysitis. The most common sites in relation to the lower
extremity are:
 tibial tubercle at the attachment of the patellar tendon (Osgood-Schlatters disease)
 calcaneous at the attachment of the Achilles tendon (Severs disease)
 anterior inferior iliac spine at the attachment of the long head of rectus femoris muscle
 iliac crest at the attachment of abdominal muscles and the superior aspect of hip abductors
 ischial tuberosity at the attachment of the hamstrings
 anterior superior iliac spine at the attachment of the sartorius tendon
In throwing sports, the apophysis of the medial epicondyle of the humerus can be involved.

With overuse apophyseal injury, there is often localized swelling and pain with a full range of joint
motion. Resisted motion is painful.

The diagnosis is made clinically. X-rays must be taken to rule out other pathology. Treatment consists
of rest to control initial pain and gradual stretching, strengthening and introduction of activity as
governed by pain.

When considering hip and groin pain in adolescents, two other conditions are important to mention.
In younger children, particularly between the ages of 5-10, one must consider the possibilities of
Legg-Calve-Perthes disease. This is idiopathic avascular necrosis of the femoral head that initially
presents as a painful limp. The limp may be exacerbated by activity but can occur independent of
activity. There is often a generalized reduction of hip motion with particular limitation of abduction
4 - Injuries to the Musculoskeletal

and external rotation. X-ray examination supports the diagnosis depending on the stage of the disease.

Management of Legg-Calve-Perthes disease is complex with serious sequelae if not well treated. An
orthopedic surgeon should carry it out.
System

The other condition that warrants mention in adolescent athletes is that of slipped femoral capital
epiphysis. This is not a common cause of hip or knee pain but may present in the pre-adolescent and
adolescent age group. This displacement of the epiphysis is medial and posterior and, hence, the
patient presents with an externally rotated and slightly flexed leg. It can occur gradually or suddenly,
and the athlete is usually male and may be slightly obese. X-ray evaluation (Figure 4.38) will confirm
the diagnosis and, particularly in the early stages, include both hips. Treatment depends on the degree
of slip, and in view of the bilateral occurrence of the condition, may warrant operative intervention on
both hips. An orthopedic surgeon should undertake the treatment process.

- 194 -
IOC Sport Medicine Manual 2000
Figure 4.38 X-ray of slipped femoral capital epiphysis.

A hamstring avulsion injury is treated initially with crutches and ice for about 10 days. Physiotherapy
rehabilitation should commence after the initial pain had settled. Initial treatment involves gentle
non-weight-bearing stretching of the hamstring muscle (Figure 4.39), stationary cycling and
interferential current. A progressive and pain-free strengthening programme involving hamstring tubing
exercises (Figure 4.39), core stability training, proprioception, and EMS should also be initiated early
in the rehabilitation plan (typically around the 2-3 week time frame). Provided they do not cause pain,
non-weight-bearing aerobic activities such as swimming and deep water running can be introduced to
maintain aerobic fitness. Advanced plyometric exercises incorporating eccentric loading, must also
be considered in the later stages of rehabilitation. A progressive activity plan incorporating a walk-run
programme and sprint exercises should be introduced before the athlete returns to heavy training and
competition.

4 - Injuries to the Musculoskeletal


System
Figure 4.39 Non-weight-bearing stretch of hamstrings - maintain neutral torso, pelvis and hip
position with relaxed foot and extended knee.

- 195 -
IOC Sport Medicine Manual 2000
Figure 4.40 Hamstring tubing exercises.

The hurdler returned to full weight-bearing at the six-week time frame, progressing rapidly from
walking to jogging to running. He was able to participate in full training thirteen weeks following the
injury.

3. Hip Pointer

Case History - Hip Pointer

A 23-year-old rugby player was in a collision with an opponent and received a knee in the left
hip area. He felt immediate pain but was able to continue for another few minutes before leaving
the field with an obvious limp. He then iced the injured area. The next morning he was in extreme
discomfort when getting out of bed and walking down stairs. Coughing and laughing exacerbated
his pain.

Physical examination revealed a fit young man having difficulty in walking and in obvious
pain. There was diffuse swelling over the anterolateral iliac crest region. Hip range of motion was
decreased due to pain in all directions. There was extreme pain on resisted trunk flexion and hip
flexion as well as some pain on abduction, adduction and hip extension. There was marked local
4 - Injuries to the Musculoskeletal

tenderness along the anterolateral iliac crest.

Discussion
Differential diagnoses of direct trauma to the iliac region of the pelvis include:
a. fracture of the iliac crest
System

b. avulsion injury
c. hip pointer

The iliac crest is an extremely sturdy bone and, as such, is infrequently fractured in athletic endeavour
unless major direct trauma is involved. An x-ray is helpful in differentiation between a hip pointer and
underlying iliac crest fracture.

Usually the mechanism of avulsion injuries is different, with the avulsion being an internal, indirect
type where, in the absence of a collision, the athlete sustains acute hip pain while doing an explosive,
powerful movement.

X-ray of the rugby players pelvis and hip was normal. Based on the history, the physical examination
and negative x-ray, the diagnosis was hip pointer.

- 196 -
IOC Sport Medicine Manual 2000
A hip pointer is an example of an external or direct traumatic injury where there is a direct blow to the
anterolateral iliac crest area. The contusion involves the sensitive periosteum along the iliac crest plus
the attachments of the abdominal muscles above and leg abductors below.

The first 7-10 days of the initial management strategy of a hip pointer injury consists of activity
reduction and the application of ice and use of non-steroidal anti-inflammatories. Physiotherapy should
commence as soon as possible. Initial physiotherapy treatment would include modalities for pain
relief and healing (ultrasound, interferential current), gentle stretching of the trunk and lateral hip
muscles (Figures 4.41), and light aerobic exercise (stationary cycling, swimming). A typical return to
activity time frame using a protective pad over the sensitive area is around 2-3 weeks.

Figure 4.41 Gentle stretching of trunk and lateral hip muscles.

The rugby player was started on the above programme. His gait normalized over seven days. Full
range of motion was achieved within two weeks. He returned to playing rugby with protective padding
over the involved area at three weeks.

4. Quadriceps Contusion (Myositis Ossificans)

Case History - Quadriceps Contusion

4 - Injuries to the Musculoskeletal


In the first half of a semi-final womens basketball game, a 19-year-old guard sustained a
direct blow to her left anterolateral thigh area from an opponents knee. She continued to play
although in the latter part of the fourth quarter she was limping due to pain and stiffness in the
area. Her team won the semifinal and they advanced to the final game that was played the next
day.

System
After the initial game, she iced the injured area and applied a tensor bandage. She had trouble
bending her knee, walking, and going up and down stairs because of discomfort and stiffness. The
following morning her leg was more uncomfortable but after gentle stretching and warming of the
area she was able to play. However, by the second quarter, she was forced to withdraw because of
severe pain and swelling in her thigh.

The next day, she limped, her left thigh was very swollen and she was unable to bend her knee
because of the pain. There was a 3 cm increase in girth on the affected side as compared to the
unaffected side. Range of knee motion showed a 5 extensor lag and only 75 knee flexion. There
was marked tenderness on direct palpation over the anterolateral aspect of the thigh with diffuse
tightness within the quadriceps muscle itself. There was no evidence of medial hip involvement.

- 197 -
IOC Sport Medicine Manual 2000
Discussion
Differential diagnoses of mid-thigh swelling and pain include:
a. overuse
b. quadriceps strain
c. femoral shaft stress fracture
d. traumatic intrinsic quadriceps strain
e. traumatic extrinsic quadriceps contusion (with perhaps progression to myositis ossificans)
f. non-athletic musculoskeletal problems - eg. neoplasia

Overuse quadriceps injuries are not common. Muscle pain is often localized to the anterior thigh and
not associated with the presence of any mass. Differentiation, however, from a femoral shaft stress
fracture is important. This can be confirmed by x-ray and more commonly, a radionuclide scan. If a
femoral shaft stress fracture is confirmed, positive steps must be taken to allow healing of the stress
fracture before return to full activity. Complete fracture of the femoral shaft can occur if this is
unrecognized.

Traumatic quadriceps strains, as opposed to overuse injuries, can occur either from intrinsic or extrinsic
means. Acute intrinsic quadriceps strain usually results in disruption of the rectus femoris. The injury
can be significant with a muscle defect and a shortened, bunched up, torn muscle palpable on the
proximal aspect of the defect.

Extrinsic quadriceps injury is a result of a direct blow or collision. The underlying muscle is bruised.
The concern with extrinsic injury is the degree of injury and the propensity for the development of
myositis ossificans, which involves calcification of the haematoma within the muscle itself.

Evidence of myositis ossificans consists of:


 large, localized, firm swelling in the quadriceps muscle mass
 significant reduction of range of motion in the hip and knee joint with the initial injury and
failure of improvement
 protracted healing pattern
4 - Injuries to the Musculoskeletal

The diagnosis of myositis ossificans is confirmed by a soft tissue x-ray (Figure 4.42) done approximately
four weeks following injury. This will show the presence of faint calcification within the haematoma
mass. A radionuclide scan can usually determine this condition 1-2 weeks post-injury. If an adequate
history of previous trauma has not been provided, the x-ray appearance of early myositis ossificans
can mimic a more sinister condition such as osteogenic sarcoma. Ultrasound imaging will detect the
System

calcification of myositis ossificans earlier then x-ray and may detect a discrete intramuscular
haematoma.

- 198 -
IOC Sport Medicine Manual 2000
Figure 4.42 X-ray of myositis ossificans involving the anterior thigh.

To minimize the likelihood of myositis ossificans, treatment must be extremely cautious. It should
include aspiration of a discrete haematoma if present. The initial phase should allow for settling of the
haematoma and prevention of aggravation. Initially one attempts to prevent further bleeding by icing
the area for 15 minutes every hour. Rest, elevation and avoidance of alcohol are important. A
compression bandage should also be used throughout the day and removed at night. As the initial pain
begins to settle (2-3 days), the gentle range of motion exercises (wall slides, stationary cycling, pool
activities) are commenced. Physiotherapy modalities (pulsed ultrasound and gentle electrical muscle
stimulation) may also be implemented at this time. Gentle stretching of the quadriceps muscle (Figure
4.43) can be added as the athlete improves. Aggressive weight-bearing activities and strengthening
exercises should not be introduced until there is full joint range of motion. In rare circumstances, if a
large mature ossified mass remains, surgery can be performed to excise it.

4 - Injuries to the Musculoskeletal


..

System
Figure 4.43 Gentle right quadriceps stretching - maintain neutral torso position with active assisted
or passively flexed knee and extended hip.

The 19-year-old basketball player sustained a quadriceps contusion and made the judgement error of
continuing to be active on a swollen, stiff thigh. The player went on to develop myositis ossificans
within the quadriceps lesion. Night and rest pain were present for five weeks. X-ray examination at 12
weeks revealed the bony mass was maturing. Adequate range of motion was achieved by 13 weeks.
Running commenced at 16 weeks while full practicing resumed at 20 weeks. At that time there was
still a 10-degree loss of full knee flexion on the affected side (compared to the unaffected side). The
patient went on to regain full range of motion at nine months post-injury.

- 199 -
IOC Sport Medicine Manual 2000
5. Hamstring Strain

Case History - Hamstring Strain

One of the best strikers on the national soccer team presented with a one-day history of left
posterior thigh pain. He recounted that during a game the previous day, he was sprinting after a
ball when he had a sudden tearing sensation in his posterior thigh on the left side. He had to stop
running and was replaced. The pain, stiffness and swelling increased over the ensuing hours such
that he was unable to walk without limping. He treated his muscle with ice and a tensor bandage.
The pain was worse the next day. He had sustained a similar injury one year earlier.

On examination, he had a decreased range of motion of the left hip because of pain in the
posterior thigh. Straight leg raising was markedly reduced. There was tenderness and fullness on
the medial aspect of the mid-posterior thigh and pain on resisted knee flexion. Stretching of the
hamstring was extremely painful. Neurovascular examination of the leg and foot was within normal
limits with normal power, tone and sensation.

Discussion
Differential diagnoses of posterior thigh pain in a fit young athlete include:
a. hamstring strain
b. sciatica
c. femoral stress fracture

The presence of tenderness in the hamstring with a palpable swelling and a normal distal neurological
examination in an acute injury setting is highly suggestive of hamstring strain.

It is extremely important to differentiate between sciatica with a normal neurological examination,


and femoral stress fracture with no pain on stressing the femoral shaft itself. If one is still unable to
make the diagnosis, then x-ray and a radionuclide scan may be performed to further delineate the
problem.

The etiology of hamstring strain is uncertain but appears to be related to:


 inadequate warm-up
4 - Injuries to the Musculoskeletal

 hamstring inflexibility
 poor hamstring strength resulting in a strength imbalance between the two hamstrings and
between the quadriceps and hamstrings
 leg length discrepancy
System

 back pain: with its origin on the pelvis, the hamstring is prone to reflex muscle tightness and
spasm in the presence of mechanical low back pain; if a muscle with increased tension has
athletic demands placed on it, then it is more prone to strain
 overuse: too much too soon; commencing training with sprint activity will place an excessive
load on the muscle and make it prone to strain
 weak abdominal muscles
The treatment of a hamstring strain depends on the grade of the lesion, the site of the lesion and the
sport of the injured athlete.

Initial management of hamstring strain consists of ice, compression bandaging, activity reduction and
the use of non-steroidal anti-inflammatory medication. Physiotherapy treatment should commence as
early as possible. Treatment would include therapeutic modalities such as ultrasound and electrical

- 200 -
IOC Sport Medicine Manual 2000
muscle stimulation. Assessment and correction of any foot, hip, pelvic or lower back dysfunction as
well as gentle massage should also be administered. Non-weight-bearing aerobic activities such as
cycling, swimming and deep water running should be introduced as soon as possible to maintain
aerobic fitness. As healing continues, progressive strengthening exercises for the hamstring muscle
should commence. Such exercises would include hamstring-tubing exercises (Figure 4.40) and
hamstring flicks (Figure 4.44) and progress to eccentric hamstring loading. A progressive hamstring
running programme should also be included. Prior to the athlete returning to full activity, he/she
should be evaluated on an isokinetic dynamometer to ensure a proper 3:2 quadriceps to hamstring
strength ratio.

Figure 4.44 Hamstring flicks - while maintaining neutral torso and hip position, briskly move into
varying degrees of knee extension.

In a Grade III strain with complete muscle rupture, particularly near the musculo-tendinous junction
in an elite athlete, the possibilities of surgical repair of the muscle should be entertained.

Following the therapy principles noted above, the striker from the national team went on to heal fully

4 - Injuries to the Musculoskeletal


from the hamstring strain in 10 weeks.

6. Femoral Fracture

System
Case History - Femoral Fracture

A 23-year-old alpine ski racer lost control on a steep, high speed turn and crashed into a
barrier, striking his right thigh directly in the mid-thigh area. He was immediately aware of a loud
crack in the area with pain, deformity and immediate inability to bear weight.

When assessed by the ski patrol, he was noted to have an angulated thigh area with his extremity
shortened and externally rotated. Distal neurovascular examination was normal and there appeared
to be no major injury to the knee joint itself. His skis were removed and he was splinted prior to his
removal by stretcher to a first aid facility. Further assessment revealed a fractured femur. He was
transported to a local medical facility where he was x-rayed.

- 201 -
IOC Sport Medicine Manual 2000
Discussion
Differential diagnoses include:
a. anterior dislocation of the hip
b. posterior dislocation of the hip
c. femoral mid-shaft fracture

The differential diagnosis of a shortened, externally rotated lower limb following major trauma is
either an abnormality in relation to the hip area or an abnormality in the mid-thigh. Anterior dislocation
of the hip will result in a shortened, externally rotated, flexed hip with marked pain on attempted
motion. There is not, however, deformity present in the mid-thigh region. If the hip is dislocated
posteriorly, then the leg will be shortened, flexed, internally rotated and adducted, any attempt at
motion of the hip will produce pain.

It is important in the initial assessment of these injuries to gain adequate x-ray examination to try and
confirm the diagnosis and to rule out the presence of a fracture around the hip joint itself or in the
region of the femoral neck.

The short-term problems associated with a dislocated hip are pain and dysfunction. The long-term
problems are those associated with avascular necrosis of the femoral head and the delay in repairing
the dislocation.

If significant injuries such as a dislocated hip or fractured femur are diagnosed, then it is essential that
the medical staff seek rapid transfer of the patient to a facility where it will be possible to perform
adequate x-ray examination and where a more definitive treatment can be undertaken.

The limb must be splinted until definitive open or closed treatment of the fracture can be carried out.
As with all fractures, one must be careful to check for any neurovascular damage, and to ensure that
no further injury occurs during the initial phase of the treatment and transport.

X-rays examination indicated that our skier had a transverse midshaft fracture of his right femur. He
was transferred to the hospital where he was treated with surgical closed intramedullary nailing. This
allowed rapid mobilization of the thigh with non-weight-bearing activity initially.
4 - Injuries to the Musculoskeletal

Rehabilitation treatment post-surgery should commence with progressive weight-bearing once the
fracture site shows evidence of good fracture healing, typically around the 6-week time frame. The
rehabilitation protocol should also include wall slides (Figure 4.45 and Figure 4.46b) and pain-free
System

isometric quadriceps contractions. Pool workouts (swimming, pool running) may be introduced at
this time to facilitate range of motion recovery and strengthening of the thigh muscles. Light resistance
stationary cycling for increasing hip and knee range of motion is also important at this stage. Resistance
strengthening exercises should start as soon as the athlete demonstrates a reasonably pain-free gait
pattern. Exercises should include mini-squats (Figure 4.46a), hamstring curls (Figure 4.40), calf raises
(Figure 4.47), gluteal strengthening, as well as proprioception exercises. Core strengthening exercises
as well as electrical muscle stimulation to the quadriceps muscle group of the affected leg while
performing squats is important for optimum strength and function recovery.

- 202 -
IOC Sport Medicine Manual 2000
Figure 4.45 Wall slides - active assisted knee range of motion.

Figure 4.46 Right leg squat progression.


a) Mini-squats - while maintaining neutral torso and pelvic position, squat on weight-

4 - Injuries to the Musculoskeletal


bearing right leg, slowly lowering the left heel to the ground, and ensuring that the
right knee is tracking over the right foot.
b) Wall slides - while maintaining neutral torso and pelvic position, apply constant
pressure into wall with the left leg and squat to desired depth on the right leg.

System

Figure 4.47 Calf raises.


- 203 -
IOC Sport Medicine Manual 2000
The skier was allowed to return to full weight-bearing activity at six weeks and skiing at twelve weeks
post-injury. He returned to full, active competition shortly thereafter. Eighteen months following his
operation, he was readmitted to the hospital for removal of the intramedullary nail. After a six-week
period to allow for remodelling of the bone around the removed nail, he returned to full, active,
unlimited activity.

F. Knee
1. Patellofemoral Pain Syndrome

Case History - Patellofemoral Pain Syndrome

A 14-year-old gymnast presented with a history of increasing pain and discomfort around her
patella. She had increased her training during the past three months, including a substantial
amount of bounding and jumping. She described the pain as an ache centred on the medial aspect
and on the under-surface of her patella. It was exacerbated by activity and eased somewhat by
rest. It was not associated with any major swelling in the knee. While she had no episodes of her
knee giving way, she was aware of a slight feeling of weakness in the joint. There was no history of
previous major patella injury although she has a sister who had an episode of patella dislocation.

Physical examination revealed an extremely muscular young woman. She had slight genu varum
in both lower extremities. She had no effusion in either knee joint. Examination of the patella
showed this to be stable but there was tenderness over the medial facet and retinacular region and
pain was elicited on patellar compression. She had a negative apprehension sign. Her quadriceps
(Q) angle was slightly increased (Figure 4.48). She had minimal patellofemoral crepitus on
active extension against resistance. The lateral retinaculum appeared slightly tight. There was no
tenderness or swelling in relation to the patellar tendon or tibial tubercle area. Ligament and
meniscal tests of the tibiofemoral joint were normal. Radiographs including tunnel view for
osteochondritis dissecans and skyline view were normal except for minor lateral patellar
subluxation.
4 - Injuries to the Musculoskeletal
System

- 204 -
IOC Sport Medicine Manual 2000
Figure 4.48 Measurement of Q angle. A large Q angle contributes to lateral patellar
subluxation.

Discussion
Differential diagnoses include:
a. patellofemoral pain syndrome
b. patellar subluxation
c. patellofemoral arthritis
d. medial plica syndrome.
e. quadriceps insertion tendonitis

4 - Injuries to the Musculoskeletal


f. patellar tendonitis

In the absence of any major effusion in the joint, or any documented history of patellar dislocation or
major episodes of giving way, patellar subluxation or dislocation is rare. Swelling of the knee joint

System
and increased crepitus may indicate chondromalacia and patellofemoral arthritis. Neither was present.

A medial synovial plica syndrome was ruled out by the absence of crepitus or snapping palpable in the
medial retinacular area with flexion and extension of the knee.

The diagnosis was patellofemoral pain syndrome.

Initial treatment was aimed at reducing the pain by the use of anti-inflammatory drugs. Physical
modalities generally include ultrasound and interferential. Some protection can be afforded the injured
extensor mechanism by reducing the impact loading. Often gentle warm-up exercises decrease the
pain during activity. Flexibility of the single-joint and two-joint quadriceps muscles can reduce pain.
Ice application immediately following exercise may reduce post-exercise aches.

- 205 -
IOC Sport Medicine Manual 2000
Patellofemoral pain is often related to a rapid increase in repetitive loading of the knee. Pre-season
fitness programmes to achieve an adequate training base and exercise capacity are critical to the
prevention of this overuse injury.

The alignment of the patella during motion within the femoral groove has been identified as another
potential source of patellofemoral pain syndrome. Vastus medialis activity can improve the normal
tracking of the patella. An active quadriceps-strengthening programme, particularly aimed at improving
the vastus medialis component, is helpful. Activation of the vastus medialis can be enhanced using
electrical muscle stimulation or biofeedback.

The use of taping or patella stabilisation sleeves can often provide more normal patella tracking and
reduce symptoms. Several special forms of taping for peripatellar pain have been developed. Taping
may also be indicative of the potential for success with stabilisation sleeves (see Figure 4.49).

Medial-lateral malalignment of the knee extensor mechanism can also be caused by pronation of the
foot, and adduction of the hip. Therefore, exercises may vary depending of the results of the clinical
examination, and may include hip stability or ankle stability exercises.
4 - Injuries to the Musculoskeletal

Figure 4.49 Patellar taping - medial glide of right patella stabilized with tape.
System

The non-operative treatment was successful with our fourteen-year-old gymnast. For a period of one
month, she limited the amount of jumping and bounding in her training, while she engaged in a
strengthening programme of the medial quadriceps using an isotonic programme. She then gradually
increased, on an alternate day basis, the volume of bounding and jumping over four more weeks, until
she had returned to a full programme.

- 206 -
IOC Sport Medicine Manual 2000
2. Iliotibial Band Friction Syndrome

Case History - Iliotibial Band Friction Syndrome

A 26-year-old, 10 km runner presented with a history of sharp, severe pain on the lateral side
of his right knee. Pain had been noticed in previous two weeks and began while he was running.
Any attempt to run through the pain greatly increased the intensity, forcing him to walk. The pain
was made worse, and came on sooner, when running downhill but gradually subsided one to two
hours after cessation of running. The runner had recently increased his weekly mileage from 60 to
100 km, and over the three weeks before the onset of symptoms had been running more hills. There
was no associated swelling, locking or symptoms of instability and the runner had no history of
trauma.

On examination, the patient, standing erect, had a slight varus alignment of both lower
extremities. General knee examination included measurement of passive range of motion, testing
for synovial effusion, testing for ligamentous stability, palpation of tendons and joint lines, and
examination for meniscal tears. No abnormalities were found during this examination except for
a point of maximal tenderness directly over the lateral femoral epicondyle. Pain was exacerbated
by direct pressure over this site and over the iliotibial band proximally, particularly when the knee
was flexed and extended. Some soft tissue crepitus and minor local swelling was noted over the
lateral epicondylar region. A one-legged squat produced pain in the right lateral knee region.
Iliotibial band flexibility was tested in a side lying position with the knee flexed and the hip extended
(Figure 4.50).

4 - Injuries to the Musculoskeletal


System

Figure 4.50 Left tensor fascia lata stretch - while maintaining neutral torso and pelvis, externally
rotate, adduct and extend left hip and leg.

Discussion
In the absence of discrete trauma, differential diagnoses include:
a. iliotibial band friction syndrome
b. early lateral meniscal cyst
c. lateral meniscus tear
d. patellofemoral pain syndrome
e. lateral compartment chondromalacia

- 207 -
IOC Sport Medicine Manual 2000
Iliotibial band friction syndrome commonly occurs in runners with a varus rather than valgus alignment
of their lower extremities, and particularly while attempting to increase running mileage. The pain is
located slightly above the joint line and is centred over the lateral femoral epicondyle. The etiology is
presumed to be due to the back and forth action of a relatively inflexible iliotibial band over the lateral
femoral condyle. It is uncommon in sports other than running and usually occurs with a rapid change
in workout intensity. It appears that reduced flexibility associated with high mileage is a contributing
factor, but the sudden onset indicates the presence of a threshold effect. In this regard, running at
distances below this threshold often fails to reproduce symptoms. Runners often report recurrent
injuries, perhaps an indication of predisposing factors such as repeating injurious training cycles,
inadequate warm-up and flexibility, or predisposing bony architecture and alignment.

If the pain is due to an early lateral meniscal cyst, it is located in the mid-portion of the lateral joint
line. The pain can be reproduced by the examiner when attempting to trap the lateral meniscus between
the joint surfaces by applying initially a varus and then valgus force while holding the meniscus
within the joint itself. A lateral meniscal cyst may be associated with a tear in the lateral meniscus.
The presence of an effusion and pain within the joint, together with the findings of an area of swelling
at the joint line, often indicates an associated meniscal tear. If a meniscal tear is present, it is treated
surgically rather than the cyst itself. Cyst excision without treatment of the meniscal tear would usually
lead to recurrence of the cyst.

The treatment of iliotibial band friction syndrome initially involves reduction of the acute inflammation
through the use of rest, modified activity (cycling, swimming), anti-inflammatory medication, and
ice post-exercise. Observational assessment of the lower extremity during running and walking can
draw attention to increased hip adduction or foot pronation that may predispose the runner to iliotibial
band friction syndrome. Stretching of the tensor fascia lata-iliotibial band would be used extensively.
Ultrasound and massage are often added to the treatment regime in the initial stages.

A graduated return to a modified activity programme is essential to prevent cyclical recurrence.


Modifications in the training schedule should allow running to the onset of pain. Intermittent rather
than constant running speeds should be used. Hills should be avoided.
4 - Injuries to the Musculoskeletal

As part of the rehabilitation and prevention regime, stabilisation exercises for the hip are often
recommended. Hip adduction during the stance phase of running is believed cause increased
compressive forces over the lateral femoral epicondyle. Exercises for the hip rotators, abductors, and
gluteus medius are recommended in these cases.
System

If these conservative approaches do not produce relief, and adventitious bursa is present deep to the
iliotibial band, injecting the bursa with corticosteroid may be valuable. Should the problem become
chronic or recurrent, then surgical release of the iliotibial band, particularly in the area of the major
irritation, may be helpful.

The 10 km runner used swimming pool running to maintain his fitness. After three weeks, he was
permitted to begin a running programme, initially up to 20 minutes per day and then increasing by
five minutes a day. He was back to his pre-injury training level and was pain-free within six weeks.
He used an intermittent pattern of running to increase his mileage above 40 km and was successful in
achieving weekly mileage of 100 km without pain by paying strict attention to stretching and by the
avoidance of hills.

- 208 -
IOC Sport Medicine Manual 2000
3. Acute Non-contact Knee Injury with Haemarthrosis

Case History - Acute Non-contact Knee Injury with Haemarthrosis

When planting her right foot to move to the left during a soccer game, our player had a sudden
severe pain in the right knee and collapsed to the ground. She was unable to bear weight on the leg
due to knee pain. Considerable swelling occurred within 6 hours. Later the knee felt wobbly when
trying to bear weight. She denied previous history of knee problem apart from cuts and abrasions.

Examination revealed swelling of the entire knee with motion limited 10 to 85 (see Figure
4.2). Palpation revealed tenderness along both tibiofemoral joint lines. Stress testing was somewhat
difficult due to pain and spasm. Posterior drawer test was negative; collateral ligament testing
revealed good end-points at 30 to varus and valgus stress. Anterior drawer test at 85 was negative
(Figure 4.3), but for Lachmans test at 30 (Figure 4.4), the knee had a poor end-point and increased
laxity. Meniscal stress testing could not be done due to pain. Circulation, sensation and muscle
testing in the leg and foot were normal.

Plain radiographs did not reveal any tibial eminence fracture, nor evidence of osteochondral
fractures, but did show a small fracture with minimal displacement at the anterolateral tibial
margin, inferior to the knee joint.

Discussion
Differential diagnoses include:
a. ACL sprain
b. MCL sprain
c. Medial or lateral meniscal tear

The sudden severe pain with a giving way of a knee with early swelling has a 60% chance, on history
alone, of being due to an anterior cruciate ligament tear. The restricted motion of our soccer players
knee may be due to micro-injury to the surrounding capsule. There is a small chance of a displaced
meniscus being responsible for the flexion contracture of 10, but if restoration of extension progresses
reasonably over two weeks of rehabilitation, then a displaced meniscal tear is less likely. The reliability
of the anterior drawer test at 90 is only 60% for detecting ACL tear, whereas at 30, the reliability is

4 - Injuries to the Musculoskeletal


over 95% if the test is positive, or even equivocal. Displacement of the tibia relative to the femur at
the time of injury may be sufficient to impact on the popliteal artery or nerves. Always ensure that
there is circulation and nerve function distal to an injury. Other possible diagnoses include: chondral
damage, meniscal tears, and minor medial collateral ligament tear.

System
This is a devastating sport injury. The knee must be protected carefully during the initial phases to
prevent a second subluxation or dislocation that can occur unexpectedly with simple activities such as
descending a step or turning while walking. Such secondary injuries often tear a meniscus enough to
displace it, truly lock the knee and require urgent surgery.

Radiographs should be taken to rule out any bony injury such as tibial spine avulsion or osteochondral
fracture, which should be surgically repaired. If a tibial eminence fracture was seen on radiographs,
then early surgical repair would be indicated because the ACL itself may be simply detached from the
tibia. Osteochondral fractures should be further evaluated with an arthroscopy to excise or repair as
needed. The anterolateral tibial margin minor avulsion fracture is associated with a complete ACL
tear. This fracture itself needs no treatment; the treatment is directed to the ACL problem. Initial
treatment is for pain relief and to minimize swelling. Crutches are usually required. Then, within a
few days, a rehabilitation programme is initiated.

- 209 -
IOC Sport Medicine Manual 2000
Active exercises are used in combination with modalities to improve range of motion, to maintain
strength, and to educate the patient. Gentle motion exercises, three times per day, within the athletes
tolerance, are recommended. Interferential therapy, elevation of the limb, ice, and decreased weight-
bearing assist with reduction of knee joint effusion.

Dramatic atrophy of the quadriceps muscle can occur within two weeks, therefore measures to control
loss of muscle mass should start early. Biofeedback, muscle stimulation, and active quadriceps muscle
exercise should be performed with compressive forces on the knee joint through standing or leg press.
Since the quadriceps muscle can produce excess motion of the tibia on the femur in the ACL-injured
athlete, the athlete should be cautioned against leg extension exercise in a non-weight-bearing position.

Figure 4.51 Unilateral leg press on shuttle/leg press.

As the athletes exercise tolerance increases, balance (proprioceptive) exercises are introduced. The
injured athlete often reports a loss of one-legged balance, therefore, progressive challenges are applied
through balance boards and soft surfaces, with or without visual cues.

Increased range of motion in the knee may not change the functional range of motion during gait.
Encouraging the athlete to use a normal gait pattern can assist with the return of normal function.

Surgical reconstruction of the injured ligament may be indicated in the athlete to return to her jumping
4 - Injuries to the Musculoskeletal

or pivoting sport of soccer. Surgery should be postponed until some reasonable restoration of motion
has been achieved so as to minimize permanent stiffness. Reconstruction of her ACL was done using
one of her own knee tendons, such as the semitendinosis or 1/3 of the patellar tendon. Protection (eg.
elastic bandage, tape or brace) and rehabilitation was undertaken. She was allowed to return to soccer
System

six months from surgery. Non-operative treatment with rehabilitation and the use of a special knee
orthosis (brace) for return to sport is often successful for non-jumping non-pivoting sports.

- 210 -
IOC Sport Medicine Manual 2000
4. Acute Knee Injury

Case History - Acute Knee Injury

A 24-year-old wrestler attempted to throw his opponent to the ground. As he twisted and applied
pressure in the hold, he felt the sudden sensation of his right knee going out of joint and snapping
back into place. There was a sudden feeling of weakness in the limb. The wrestler fell and was
unable to continue the match. As he attempted to step down from the ring following the match, his
knee still felt unstable and weak. There was no history of previous injury.

Examination immediately following the bout showed the presence of a slight swelling. No
peripatellar tenderness or apprehension to lateral dislocation was found. There was marked
tenderness over the proximal attachment of the medial collateral ligament in the mid-joint-line
area. Valgus stress at 0 of flexion showed no laxity, and at 30, there was a mild laxity with a poor
end point. Varus testing was within the normal limits compared to the other knee. There was no
posterior sag (Figure 4.52) and a negative posterior drawer test at 90 of knee flexion. Anterior
drawer test at 90 and at 30 of flexion had good end-points.

Although it was possible to place the knee through a range of motion from 0-120 initially, as
the swelling (most likely haemarthroses) increased during the ensuing four-hour period, the range
of knee motion became more limited with a 20 degree loss of full extension, and flexion to 90.

Discussion
Differential diagnoses include:
a. medial collateral ligament (MCL) sprain grade 3/3
b. medial meniscus tear
c. anterior cruciate ligament tear (ACL)

If there is a reasonably large haemarthrosis, it is almost certain that the MCL tear is associated with an
injury to the ACL. Localized swelling over the MCL occurs with an injury to the superficial fibres of
the MCL; the deep fibres are often torn, causing grade 2 instability with no swelling whatsoever.

The feeling of the knee going out of joint is significant. The examination finding of no laxity in

4 - Injuries to the Musculoskeletal


extension effectively tests the posterior capsule. The maximum tenderness was at the MCL origin on
the femoral epicondyle rather than on the meniscus and this finding helps rule out meniscal tear
(which is rare with a valgus knee injury). In a slightly flexed position (posterior capsule loose), the
valgus stress is applied to the MCL itself and our finding was excess laxity, indicating a significant

System
tear of the MCL (Figure 4.5).

Management of complete MCL tear is generally non-operative. Secure splinting or bracing allows
early range of motion in flexion and extension but not valgus angulation of the knee. External rotation
of the knee should also be avoided. The athlete should wear this protection for 6 weeks throughout the
day and night. It may be removed in a safe environment for flexion-extension range of motion exercises
or stationary bicycling. Gentle testing of motion is undertaken to ensure that extension to zero degrees
as well as flexion past 100 is being obtained by 4 weeks to avoid permanent contracture. Gentle
stressing of the MCL for its endpoint is done also. If the ligament is tightening well, then more
freedom can be given for motion and activity but if the endpoint remains soft then continued protection
might be appropriate for 8 to 12 weeks.

The athlete should initially emphasize range of motion exercises for knee flexion and extension within
their tolerance. While lateral motions (such as whip kick in the pool) are to be strictly avoided, the
- 211 -
IOC Sport Medicine Manual 2000
athlete can work towards any flexion-extension activity within their level of tolerance. Therefore,
cycling can be increased quickly to a level that maintains aerobic fitness. Similarly, strengthening of
the hip, thigh, and leg can be initiated early and progressed quickly if medial-lateral stresses are
eliminated. Earlier return to sport can be achieved with a good knee orthosis if the sport would allow
a metal-containing orthosis.

Pain can be a significant factor in the MCL injury, inhibiting increases in range of motion and producing
muscle weakness. Interferential therapy, ultrasound, protected weight-bearing and ice might add some
additional benefit to pain medications.

It is not until sufficient healing has occurred and clinical tests indicate that the ligament has regained
its role in stabilizing the knee, that cutting maneuvers, twisting, jumps, or uneven ground should be
attempted.

The wrestler completed a protection and rehabilitation programme and by three months had good
knee motion and a solid end-point to valgus stress, indicating proper healing of the MCL. He returned
to competition 6 weeks later.

Figure 4.52 Positive posterior tibial sag test a) traditional test view b) non-traditional view showing
deformity. Arrow indicates point of posterior sag of proximal tibia.
4 - Injuries to the Musculoskeletal

G. Lower Leg Pain


System

Case History - Lower Leg Pain

A 25-year-old female Olympic middle distance medallist complained of a right medial lower
leg pain while running. She had been participating in the European racing circuit and was training
extensively on pavement. At home, she usually trained on trails and flat fields. Her quality running
shoes were two months old and in good condition. The onset of her pain was gradual, increased by
running, and partially relieved by rest. On her last run, she was forced to stop at 2 km and to walk
home. That night she was unable to sleep because of leg pain which was dull and aching. She
noted minor swelling over the medial tibial border. She was amenorrheic.

Discussion
In the past, athletes with lower leg pain often referred to this symptom as shin splints. This is a non-
specific term and the physician should be able to distinguish the various pathological conditions that
cause lower leg pain in the athlete.

- 212 -
IOC Sport Medicine Manual 2000
Differential diagnoses include:
a. popliteal artery and /or peripheral nerve entrapment
b. chronic exertional compartment syndrome
c. periostitis of the tibia
d. stress fracture of the tibia

Although less common than other causes, artery entrapment syndromes must be considered when the
athlete complains of lower leg pain with these characteristics:
 a history of intermittent claudication during walking or running
 physical findings of reduced pedal pulses with sustained active plantar flexion or passive
dorsiflexion of the foot
 abnormal Doppler and/or angiographic studies done under exercise stress
Vague or bizarre complaints of pain in the lower leg and feet by the athlete should not be dismissed
without considering popliteal artery entrapment. Claudication calf and foot pain can be accompanied
by paraesthesia, blanching, and coolness. The cause is compression of the popliteal artery either by
the origin of the medial head of gastrocnemius or plantaris muscle or by an aberrant fibrous band.

4 - Injuries to the Musculoskeletal


Figure 4.53 Arteriogram of popliteal artery entrapment.

System
Chronic exertional compartment syndrome is a frequent cause of lower leg pain or tightness,
particularly in long-distance runners. The pain may be associated with muscle weakness and altered
lower leg sensation. The resting compartment pressure in these patients may be elevated (> 15 mmHg).
This results from the combination of muscle hypertrophy and interstitial fluid accumulation secondary
to muscle injury and vascular dysfunction. When combined with the normal increase in intra-
compartmental blood volume during exercise, blood flow becomes compromised, leading to myoneural
ischemia. Rest relieves symptoms but exercise triggers the process at the next exposure and a chronic
pattern is established. Some athletes experience a second day phenomenon, in which the amount of
exercise that can be performed without pain one day is less than the previous day. The specific
compartment involved would dictate the specific site of muscle weakness or paraesthesia. The four
sites are the anterior, lateral, superficial and deep posterior compartments of the leg.

- 213 -
IOC Sport Medicine Manual 2000
Figure 4.54 Cross section of lower leg compartments. The muscles are enclosed in four well-
defined compartments.

Diagnosis of chronic anterolateral compartment syndrome is continued by pressure measurements


done after symptom provocation with repeated ankle dorsi- and plantar flexion in the supine position.
Repeated heel raises are used to provoke posterior compartment syndrom. An initial pressure exceeding
30 mmHg that slowly decays over the following 5 minutes is strongly suggestive of chronic
compartment syndrome. There is little risk of vascular or neurologic injury to the patient during
assessment of the anterior, lateral, or superficial posterior compartments. However, pressure
measurement of the deep posterior compartment may be complicated by injury to the tibial nerve and
posterior tibial vasculature. Therefore, the catheter insertion should not be done blindly but under
ultra-sound visualization. Initial treatment may consist of activity modification, physiotherapy soft
tissue techniques/modalities and/or taping support/orthotics. However, the most reliable treatment of
chronic compartment syndrome is a fasciectomy.

Our Olympic medallist in the middle distances had pain and swelling over the medial tibial border at
the junction of middle and lower thirds. The tibia was more tender on the right than on the left and the
4 - Injuries to the Musculoskeletal

pain was increased by single leg hopping. X-ray examination was normal but a radionuclide scan
showed focal uptake across the full thickness of the tibia at that level. The diagnosis was tibial stress
fracture. Periostitis would show diffuse linear uptake through a section of the tibia and would be
treated in a similar manner as that of a stress fracture.
System

Treatment involved modification of training. Initially, weight-bearing was restricted (eg. no running
or jumping) and training was changed to running in deep water, swimming and cycling. A long air
bladder-type ankle brace (eg. 16" Aircast stirrup ankle brace), worn as much as possible, is useful for
decreasing the time required for pain resolution. When pain was absent for 14 days, the runner resumed
her training in a progressive and graded manner. An example that may be followed is a walk-run
programme (see Table 4.4). The duration and intensity of training was then gradually increased to
usual levels. The recovery from a tibial stress fracture could take up to 12 weeks. Our Olympic runner
resumed racing nine weeks after diagnosis and treatment.

- 214 -
IOC Sport Medicine Manual 2000
Figure 4.55 X-ray of tibial stress fracture.

H. Ankle and Foot

1. Ankle

Case History - Ankle

A 15-year-old elite tennis player twisted his right ankle in a depression on the clay court surface
on which he was practicing. He experienced immediate pain and swelling on the medial and
lateral aspects of the ankle. There was maximal tenderness over the anterior talofibular ligament.
He was unable to bear any weight on his right leg. There was no history of a previous injury to the
right ankle.

Discussion

4 - Injuries to the Musculoskeletal


Differential diagnoses include:
a. fractures of the tibia or fibula
b. osteochondral fracture of the talus
c. subluxation of the peroneal tendons

System
d. ankle inversion sprain
e. subtalar joint/mid foot joint(s) sprain
f. strain of peroneal muscles
g. injury to peroneal nerve

Expeditious management of this situation is dependent on proper diagnosis and then initiation of
aggressive treatment. Several conditions may mimic the classic ankle inversion sprain.

At the local emergency hospital, x-ray examination did not reveal evidence of any fracture. Fractures
of the medial or lateral malleoli may be immediately evident on initial x-ray examination. In some
situations, however, trauma may trigger a disturbance in bone remodelling which causes persistent
pain. X-rays may remain normal even after several weeks of disability. Technetium scintigraphy will
often demonstrate focal uptake over the lateral or medial malleolus, confirming the physicians clinical
suspicion.

- 215 -
IOC Sport Medicine Manual 2000
Continued pain in the ankle joint beyond 2-3 weeks post-injury may indicate an osteochondral fracture
of the talus. The patient will usually complain of pain on weight-bearing and perhaps persistent swelling
of the ankle. X-ray examination may be normal but technetium scanning will demonstrate increased
uptake in the talus. CT or MRI may show details of a displaced or free-floating lesion as a loose body
of cartilage and bone or just cartilage. An undisplaced osteochondral fracture may respond to
conservative treatment, but a loose body may require removal and drilling of the cavity. If scans are
normal, the cause of pain may originate from superficial synovial irritation.

Subluxation of the peroneal tendons may be confused with an ankle sprain. The retinaculum holding
the peroneal tendons in the groove below the lateral malleolus may be torn and the peroneal tendons
may be dislocated from their normal position to lie superficial and anterior on the surface of the lateral
malleolus. This is often associated with a snapping sensation and may be repetitive. It causes pain
and swelling over the lateral ankle and is a common injury in skiers.

Diagnosis of a subluxed peroneal tendon is made by suspicion from the history combined with
displacement plus or minus pain from the tendons with resisted eversion of the ankle.

Peroneus
Longus

Fibula

Peroneus Brevis

Superior Inferior
Peroneal Peroneal
4 - Injuries to the Musculoskeletal

Retinaculum Retinaculum

Figure 4.56 Anatomy of the peroneal tendons.


System

If pain and disability persists, the treatment is surgical reconstruction of the retinaculum holding the
peroneal tendons under the lateral malleolus.

The athlete may also have strained muscles with the injury (eg. peroneal muscles), evident by pain/
weakness on resisted testing, and by bruising and swelling of the lateral calf and ankle.

Injury to the common peroneal nerve or its branches can occur secondarily to stretch of the nerves, as
well as with swelling or bleeding in the area. Diagnosis is via signs and symptoms, which include the
reported pain distribution, peroneal nerve innervated muscle weakness, Tinels sign, altered nerve
length-tension relationships (neuromeningeal mobility see Unit 11 - Rehabilitation).

Treatment is by local anti-inflammatory modalities and nerve mobility exercises, as indicated by the
examination.

- 216 -
IOC Sport Medicine Manual 2000
With inversion ankle sprains, the anterior talofibular ligament (ATF) is most frequently injured. More
severe injuries may involve the calcaneofibular ligament (CF), posterior talofibular ligament (PTF),
anterior inferior tibiofibular ligament (AITF), interosseous membrane, and ligaments of the subtalar
and calcaneocuboid joint. Physical examination with the anterior drawer (Figure 4.57) and talar tilt
tests (Figure 4.58) is helpful to assess the integrity of the ATF and CF ligaments, respectively. Based
on the physical examination, the severity of lateral ligament ankle inversion sprains may be classified
in three grades, as outlined in Table 4.2.

Fixation of
Posterior tibia and
translation fibula
of tibia and
fibula

Fixation of Varus
talus loading

Figure 4.57 Ankle anterior drawer. Figure 4.58 Talar tilt test - for varus
loading of calcaneofibular ligament
and inferior tibiofibular joint
ligaments.

Table 4.2 Ankle injury grading.

Physical Functional ATF


Grade CF Tear Testing

4 - Injuries to the Musculoskeletal


Findings Impairment Tear
Slight lateral
(-) anterior drawer
I swelling and none partial none
and (-) talar tilt
tenderness

System
Moderate (+) anterior
II swelling and some complete partial drawer and (-)
tenderness talar tilt
Marked (+) anterior
III swelling and complete complete complete drawer and (+)
tenderness talar tilt

X-ray examination is useful to rule out bone fractures. It is especially indicated if there is bony tenderness
on palpation of the tip and posterior aspects of the medial and lateral malleoli, the navicular, and the
base of the 5th metatarsal. The inability to weight-bear immediately after the injury and at the time of
examination are also strong indications for x-ray. Stress inversion views in the acute injury are probably
unnecessary as they are very uncomfortable for the patient and probably do not add more diagnostic
- 217 -
IOC Sport Medicine Manual 2000
information if a proper physical examination is performed. If rupture of the AITF ligament with an
interosseous membrane injury is suspected, then an ultrasound is a more appropriate investigation.

The initial management of all lateral ankle sprains is generally conservative, although surgical
reconstruction may be indicated for third degree injuries. Conservative management of the acute
lateral ankle sprain may be considered in terms of acute and rehabilitation phases. The goal of the
acute phase is protection and pain and swelling control, while the goal of the rehabilitation phase is
recovery of function.

The main aspects of acute phase of treatment may be remembered by using the PRICE acronym:
Protection, Rest, Ice, Compression, and Elevation.
 Protection - The ROM of the ankle should be restricted for comfort and to prevent further
injury to the damaged ligaments. This may be accomplished with compressive elastic bandage,
adhesive tape (initially open basket weave and then more closed techniques as swelling
resolves; see taping diagrams, Unit 11), or semi-rigid or non-rigid ankle braces. Protection
should be used during the acute and rehabilitation phases of treatment and also during the
initial return to activity period. Crutches may be used initially if the athlete is very
uncomfortable. It is best to avoid the use of plaster cast immobilization in order to prevent
excessive muscle atrophy and prolonged ankle stiffness.
 Rest - Relative rest and cross training may be started as symptoms permit in order to preserve
general conditioning of the athlete. Activities may include cycling, swimming or water running,
and modified strength training. The ankle should be protected during these activities.
 Ice - Direct application of ice or via cryocuff (or similar modalities) can be the initial methods
used after 5 days.
 Compression - Compression of the ankle for swelling control may be achieved in conjunction
with protective strategies, as suggested above.
 Elevation - To encourage resolution of swelling, the ankle should be held in a position above
the level of the heart.

Adjunctive treatment with anti-inflammatory medications and physiotherapy (modalities, mobilization


and soft tissue techniques (ie. massage), as required) is additionally useful in the acute treatment
4 - Injuries to the Musculoskeletal

phase to improve pain and swelling.

Once pain and swelling of the ankle is under control and the patient is able to tolerate further treatment
(usually within one week), then the rehabilitation phase of the treatment may be started.
System

The key components of the rehabilitation phase are recovery of ankle range of motion, strength and
proprioception.

Range of motion drills are usually begun with open kinetic chain range of motion exercises. These
should be in pain-free active mid-range of motion. Each drill should be repeated 10-30 times each
morning and evening. Examples include:
 circle foot clockwise
 circle foot counterclockwise
 flexion and extension of toes
 single plane movements: plantar flexion/dorsi flexion/inversion/eversion
Strength drills should be initiated at the same time. These could be done against the elastic resistance
of a bicycle inner tube or surgical tubing and should include: pain-free, active concentric mid range of
- 218 -
IOC Sport Medicine Manual 2000
motion; then as healing allows, progress to pain-free outer range of motion; and finally, through the
entire range of motion (concentric and eccentric):
 inversion
 eversion
 plantar flexion
 dorsiflexion
In addition to the rubber tubing strengthening drills, an eccentric heel drop strengthening drill can be
started once the athlete can weight bear without pain. The athlete stands on the balls of the feet on a
step in a stairwell, one hand on the railing, raises to the tip-toe position (Figure 4.59a) and then drops
the heel as far as it is comfortable (Figure 4.59b). Initially, this drill is done with both feet, first slowly,
then quickly. Three sets of 20 repetitions are suggested daily. As the rehabilitation continues, the drill
is gradually altered so that the heel raise and drop are performed solely by the injured leg, as shown on
Table 4.3.

Table 4.3 Heel drop programme.

Build-up
-3 sets of 20 repetitions daily
Both legs slow 7 days
Both legs quick drop, slow up 7 days
Single leg slow 7 days
Single leg quick drop, slow up 7 days
Total 28 days

4 - Injuries to the Musculoskeletal


System

Figure 4.59 a) Heel raise of right leg. b) Heel drop of right leg.

Proprioception training can started once ankle ROM and strengthening seem to be improving. The
athlete should begin with one-leg standing with arms at the side and eyes closed. The athlete attempts
to maintain balance on one leg as long as possible. Once the athlete is able to maintain their balance
for more than 60 seconds, then more challenging balance drills with a wobble or tilt board may be
incorporated. A wooden disc 50 cm in diameter is mounted on a shallow central hemispherical rocker
base with a radius of 6 cm that will allow the athlete to practice multi-directional movement. Three

- 219 -
IOC Sport Medicine Manual 2000
sets of 20 static balance: a) board parallel to floor x 20 seconds, b) side-to-side and then c) front-to-
back movements should be practiced daily. The board should not touch ground with any of the
movements but rather the athlete controls movement of board in the air. Mini trampoline, foam roller
tube, foam pad may also be used with balance drills (eg. squat single leg drills, running simulation
drills). See Unit 11 - Rehabilitation, Proprioception for more details.

All the early rehabilitation techniques do not involve running. Reintroduction of running and jumping
activity should be withheld until range of motion and strength are recovered. Pain-free and vigorous
single leg hopping is the minimal requirement as a functional test before reintroduction of running on
dry land (treadmill, track and outside).

Running can be initiated by having the athlete run on a smooth flat surface like an oval track. The
athlete starts by walking the curves and jogging the straights, both clockwise and counter-clockwise.
This is followed by jogging the curves and sprinting the straights and finally by sprinting the curves
and jogging the straights, both clockwise and counter-clockwise. Further drills can be done that include
figure eight runs (see Figure 4.60), T-drills (Figure 4.61), and other sport specific training drills.
Return to play/practice guidelines must be adhered to. This involves the athlete being pain-free, having
full range of motion and demonstrating good quality movement control on physical examination and
functional tests (hopping, squat, full squat, sprints).

Start

Figure 4.60* Running drills - Figure eight.


4 - Injuries to the Musculoskeletal

Start
System

Figure 4.61* Running drills - T-drills.

*Distances and speeds for Figure 4.60 and 4.61 can be made sport specific.

The 15-year-old elite tennis player followed this programme of rehabilitation with a specific running
drill. Starting on the base line of the tennis court, he sprinted one-half way to the net, pivoted and ran
to the base line. He then sprinted to the net, pivoted and returned to the base line. This was repeated
with appropriate rest intervals until a sense of confidence with his ankle was established. Ankle
protection with taping or bracing may be continued in the initial return-to-play period for 6-12 weeks.

- 220 -
IOC Sport Medicine Manual 2000
2. Medial Foot Pain

Case History - Medial Foot Pain

A 22-year-old 100 m runner complained of right medial foot pain. Initially, he noted the aching
pain below and anterior to the medial malleolus after training runs, particularly when using
spiked shoes on the track. Over several weeks, the intensity of the pain increased causing a limp.
He discontinued training at this point and sought medical attention.

Discussion
Medial foot pain in an athlete without a history of acute trauma but with an exposure to potential
overuse should alert the physician to the following differential diagnoses:
a. posterior tibialis, flexor hallucis longus, or flexor digitorum longus tendonitis
b. plantar fasciitis
c. tarsal tunnel syndrome
d. stress fracture of the navicular, cuneiforms or 1st metatarsal
e. symptomatic accessory navicular
f. mid foot, hind foot, or forefoot articular dysfunction

After taking an appropriate history, evaluation should include assessment of the passive and active
biomechanical alignment of the pelvis and lower extremities. The presence of genu varum or valgum,
tibial varum, pes planus and pes cavus may contribute to excess supination or pronation during the
running gait. These biomechanical conditions (even when minor) may predispose the individual to
abnormal stress when combined with excessive training loads, inadequate recovery, hard running
surfaces or shoes with minimal support.

4 - Injuries to the Musculoskeletal


System

Figure 4.62 Pes cavus, normal, and pes planus.

Posterior tibialis, flexor hallicus longus, or flexor digitorum longus tendonitis can be identified by
localized tenderness and swelling over the inferior surface of the medial malleolus. The tenderness
and swelling may extend proximally and posteriorly to the medial border of the tibia. Pain may be
precipitated by resisted active inversion of the plantar-flexed ankle for posterior tibialis, whereas
inversion, plantar and toe flexion may elicit pain if flexor hallicus longus or flexor digitorum longus
are affected. There may be associated ankle inversion weakness, increased rear foot valgus during
standing, and tight gastrocnemius and soleus muscles.
- 221 -
IOC Sport Medicine Manual 2000
In general, treatment for this condition consists of activity modification, physiotherapy soft tissue
techniques and modalities, ankle strengthening (concentric and eccentric), appropriate stretching of
leg/ankle complex muscles, and anti-inflammatories. Tape or orthotic support may be helpful especially
if the problem is recurrent and if excessive pronation and/or pes planus is present (Figure 4.63). A
trial of medial and longitudinal arch taping may be a good predictor of success with orthotics.
Equipment modification may be needed (ie. shoe wear, running surface, etc - see Unit 15 -
Biomechanics).

Figure 4.63 Taping orthotic support - medial and longitudinal arch tape support.

Plantar fasciitis can be distinguished by tenderness at the medial calcaneal tubercle with radiation
anteromedially from the heel along the long arch. Pain can be excruciating and intensified by direct
weight-bearing pressure on the heel. Symptoms may also be pronounced after the foot has been
relaxed in a plantar-flexion position for a prolonged period of time such as the first step after waking
from sleep. Initial treatment consists of activity modification with avoidance or reduction of any
aggravating activities. Stretching of the gastrocnemius, soleus, and intrinsic foot muscles numerous
times a day (Figure 4.64), followed by daily strengthening exercises of these muscles after symptoms
have improved, is the most critical component of treatment. Icing after activity and oral NSAIDs may
4 - Injuries to the Musculoskeletal

be useful in reducing symptoms. Although the benefits of physical therapy modalities are not well
studied, use of ultra sound, interferential current or laser may be of some benefit. For more chronic
symptoms, a moulded ankle-foot orthosis that holds the ankle at 5 dorsiflexion can be used during
sleep (tension night splint). This adjunctive treatment has been shown to be helpful, particularly if
System

morning symptoms are frequent. The benefits of heel pads, arch taping and in-shoe orthotics are also
not well studied, but may be tried if other measures fail to improve symptoms. If all of the above
treatments do not improve symptoms, then a cortisone injection of the plantar fascia may be attempted.
Cortisone injections should be administered by one who is familiar with injection of this site, as it
may be complicated by plantar fascia rupture, (especially if the belly of the plantar fascia is injected)
and heel fat pad atrophy. Surgical management may be considered if diligent conservative management
fails to improve symptoms after 9-12 months.

- 222 -
IOC Sport Medicine Manual 2000
Figure 4.64 Posterior calf stretching of right leg
a) gastrocnemius (knee extended) b) soleus (knee bent).

Tarsal tunnel syndrome is characterized by medial ankle and mid-foot pain, intrinsic muscle atrophy
and paresthesia caused by compression of the posterior tibial nerve or its two terminal branches beneath
the flexor retinaculum. Medial and lateral plantar nerve mobility may be altered (see Unit 11 -
Rehabilitation, Mobilization). Tarsal tunnel syndrome may at times be confused with plantar fasciitis.
Fracture or sprain of the foot or ankle, as well as tendonitis or excessive pronation, may trigger the
compression in the tarsal tunnel region. Tinels sign may be found by tapping over the tarsal tunnel
area below the medial malleolus. Electrodiagnostic tests may be helpful, but x-rays and radionuclide
scans would be normal.

Treatment is identical to the approach used for plantar fasciitis. If conservative methods are not
successful, surgical release may be necessary.

An accessory navicular may be symptomatic causing medial foot pain, but radiological examination
distinguishes it from the more threatening problem of stress injury to the navicular. Stress fracture of
the navicular cuneiform or 1st metatarsal is potentially a serious injury because of the risk of non-

4 - Injuries to the Musculoskeletal


union, avascular necrosis and osteoarthritis.

System

Figure 4.65 Plantar fasciitis.

- 223 -
IOC Sport Medicine Manual 2000
Figure 4.66 Tarsal tunnel syndrome.

Our 100 m runner was found to have neither swelling over the dorsum and the medial arch of the foot
nor tenderness localized to the proximal medial arch and dorsally over the talonavicular joint. X-rays
are often not diagnostic. A radionuclide scan is a sensitive screening test and may be required if
clinical suspicion is strong and plain radiography is negative. Computerized tomography (CT) will
demonstrate the presence of fissure cracks or displacement. Our athlete had a positive radionuclide
scan but CT scan showed no fissure crack. The diagnosis was navicular stress syndrome. This represents
a pre-stress fracture state.

The basic cause of a stress fracture is overuse. Too much running volume in terms of time and intensity
too soon in a training programme is the most frequent error. Once the diagnosis has been established,
weight-bearing activity, particularly running and jumping, should be stopped in order to allow a return
of normal bone remodelling. Usually, cessation of weight-bearing is sufficient to reduce pain.
Physiotherapy is initiated at the outset, with the goal of restoring strength to adjacent muscle groups.
This can be achieved by non-weight-bearing progressive resistance muscle training drills, augmented
by anti-inflammatory and pain relieving modalities (see Unit 11 RehabilitationModalities). Emphasis
is on eccentric training of the entire lower quadrant. Assessment of lower extremity passive and dynamic
4 - Injuries to the Musculoskeletal

biomechanics should be done and findings addressed (see Medial Foot Painsection 2 above). The
treatment plan of the fractures of the lower extremity may be outlined as follows:

Phase I - Non-weight-bearing exercise phase


System

The goal is to alter activity, so as to remove pain, yet restore local muscle strength by substituting
non-weight-bearing cardiovascular drills.
 No running or jumping is allowed.
 Cycling, in line skating, swimming or running in water is substituted to maintain cardiovascular
and muscular fitness, usually starting with 5-10 min of exercise and progressing to 40-45 min
on alternate days.
 Anti-inflammatory medication is given if pain is troublesome.
 Crutches are allowed for 1-2 days if pain is severe. Plaster cast immobilization should be
avoided when possible, as this causes further muscle atrophy, prolonging ultimate recovery.
 Muscle strengthening drills are similar to those used to rehabilitate ankle inversion injuries
(see previous Ankle and Foot 1. Ankle case history).
 Orthotic devices may be prescribed when marked biomechanical variations are observed.
Often a trial of supportive taping is used initially ie. medial and longitudinal arch support.

- 224 -
IOC Sport Medicine Manual 2000
This may be all that is required. If tape is helpful but must be used for a prolonged time then
custom made orthotics may be appropriate. Runners often adapt more readily to soft or semi-
rigid foot orthosis. Patients with marked tibial varum or subtalar or forefoot varus benefit
from some reduction in pronation; excessive pronation can produce excess rotational stress
in highly repetitive movements like running. Excess pronation can be reduced by medial
rearfoot or forefoot posting on the orthotic device. If an orthotic device is prescribed, its use
should be started during the non-running phase of treatment; an initial hour of wear can be
increased by an hour each day to allow a gradual adaptation of the muscles to the new pattern
of movement.
 Equipment modification may be needed ie. shoe wear, track, etc.

Phase II - Weight-bearing exercise phase


The goal of this phase is to gradually reintroduce ballistic, gravitational activity such as running, to
stimulate osteoblastic activity, and thus strengthen the site of the stress fracture. This phase begins
when the patient has been pain-free for two consecutive weeks. The following progression on Table
4.4 is recommended.

Table 4.4 Walk-run programme.

Week Monday Wednesday Friday

1 10 min walk 20 min walk 30 min walk


6 times (4.5 min walk + 6 times (4 min walk + 1 6 times (3.5 min walk +
2
0.5 min run) min run) 1.5 min run)
6 times (3 min walk + 2 6 times (2.5 min walk + 6 times (2 min walk + 3
3
min run) 2.5 min run) min run)
6 times (1.5 min walk + 6 times (1 min walk + 4 6 times (0.5 min walk +
4
3.5 min run) min run) 4.5 min run)
5 30 min run 30 min run 30 min run

4 - Injuries to the Musculoskeletal


*Walking or running must be pain-free.

 Once the athlete has completed some variation of the basic programme, modifications can be

System
made to the programme to make it more sport specific ie. incorporate speed into the session.
 Alternative types of training such as cycling, swimming, or running in water are continued on
every other day.
 Local muscle-strengthening drills should continue.
 Activities should also include proprioceptive training and running drills. (see previous Ankle
and Foot 1. Ankle case history).

The period spent in Phase I is typically three to six weeks, and Phase II requires about 6 weeks before
resuming training. After four weeks in Phase I and six weeks in Phase II, our 100 m runner resumed
normal training.

- 225 -
IOC Sport Medicine Manual 2000
3. Forefoot Pain

Case History - Forefoot Pain

A 24-year-old soccer player complained of right forefoot pain that was increased by playing
on a hard, dry soccer field. Initially he was aware of mild pain over the mid-forefoot just proximal
to the metatarsophalangeal area. Later he became aware of a sensation during and after playing
as if he was standing on a small pebble in his shoe. This sensation was amplified when walking
barefoot on a tile floor. Occasionally he noted a tingling or numb sensation between his toes. He
later began to limp and was forced to walk on the lateral side of the foot. After two weeks of
symptoms he sought medical aid.

Discussion
The general onset of forefoot pain in the athlete may be caused by several important disorders.

Differential diagnoses include:


a. hallux toe sesamoid stress fractures or osteochondritis
b. stress fractures of the metatarsals
c. hallux valgus
d. hallux rigidus
e. plantar wart
f. inflammatory arthritis
g. polyarthritis or gout
h. Mortons neuroma/medial plantar nerve inflammation
i. metatarsals interosseous ligament sprain

The location of the sesamoids as weight-bearing points under the head of the first metatarsal expose
them to potential injury. With great toe sesamoid stress fractures, pain develops at the first
metatarsophalangeal joint. The onset of pain may be over several days but it is occasionally more
acute. Swelling, redness and tenderness are often present over the plantar surface of the joint. In acute
cases, this can appear similar to gout or cellulitis. In chronic cases, swelling and tenderness are minimal,
but patients nevertheless complain of pain on weight-bearing, which is increased by running and
jumping, particularly at take-off. Radiographic examination has limited value, since multipartite
4 - Injuries to the Musculoskeletal

sesamoids are present in up to one-third of normal subjects. A radionuclide scan is positive in stress
fractures of the sesamoids. Treatment of this condition involves the use of an orthotic device to reduce
the load on the first metatarsophalangeal joint. If this is not successful, surgical excision of the fractured
sesamoids may be useful in achieving functional recovery. Stress fractures of the second and third
System

metatarsals are the most common stress fractures of the foot. Pain in the forefoot with swelling and
tenderness over the distal shaft and neck are early features. Diagnosis is made by radionuclide scanning.

Hallux valgus is obvious on inspection of the foot. The metatarsal angle usually exceeds 12, as the
great toe deviates towards the lesser toes. Hyperpronation and constricting narrow shoes may cause a
disproportionate weight-bearing on the great toe. Local callus and bursa develop medially over the
first metatarsal head. Treatment of hallux valgus involves review of shoe design and control of excess
pronation with tape or orthotic devices as well as biomechanical analysis of entire lower quadrant (see
Ankle and Foot 2. Medial foot pain case history). Extensive surgical procedures, such as osteotomies,
should be delayed until retirement of the elite athlete.

Hallux rigidus is characterized by a painful first metatarsophalangeal joint, reduced range of motion
(less than 20 dorsiflexion) and radiological evidence of degenerative hypertrophic arthritis. Treatment

- 226 -
IOC Sport Medicine Manual 2000
involves tape or orthosis to redistribute functional weight-bearing as well as biomechanical analysis
of entire lower quadrant (see Ankle and Foot 2. Medial foot pain case history).

Figure 4.67 Hallux valgus.

Plantar warts can create painful lesions of the forefoot in athletes (plantar warts are also discussed in
Unit 3, section J - Dermatology 2a).

Pain and swelling in the forefoot and toes which involves more than one joint and is bilateral should
raise the suspicion of polyarthritis. Investigation for a systemic rheumatic disease is then indicated.
Systemic diseases of the small joints of the feet include:
 rheumatoid disease
 inflammatory arthritis
 psoriatic arthritis
 Reiters syndrome
 ankylosing spondylitis
 osteoarthritis

4 - Injuries to the Musculoskeletal


Gout commonly affects the first great toe but can affect any part of the foot. Diagnosis should involve
history, physical examination, joint aspiration and laboratory determinations on blood samples.
Treatment involves non-steroidal anti-inflammatory agents and possibly local injection of
corticosteroids. Taping and/or orthotics may assist some patients.

System
Our soccer player with forefoot pain noted a burning sensation on the plantar surface of the foot
associated with pins and needles of the third and fourth toes. Occasionally he experienced a sudden
electric shock sensation in the forefoot. He noted relief of pain on removal of shoes. Palpation of the
forefoot revealed tenderness between the third and fourth metatarsal heads. He experienced sharp pain
on compression of the forefoot associated with a palpable click.

X-ray and radionuclide scans were normal. These findings are consistent with diagnosis of Mortons
neuroma or mid plantar nerve entrapment (see Figure 4.68). This is caused by collapse of the transverse
metatarsal arch plus or minus interosseous ligament sprain/injury and subsequent irritation and swelling
of the common digital nerves. Chronic irritations cause fibrous adhesions, thickening of the nerve and
eventually neuroma formation. Treatment should initially consist of wearing appropriate width shoe-
wear and a metatarsal pad and/or taping or orthotic support. Nerve mobility may be assessed and
addressed as indicated. A cortisone injection may be required if symptoms persist. Ultimately, if
symptoms do not resolve, surgical excision of the neuroma may be required.
- 227 -
IOC Sport Medicine Manual 2000
Irritation
and
swelling
of the
common
digital
nerve

Figure 4.68 Mortons Neuroma collapse of the transverse metatarsal arch can trap and irritate the
digital nerves.

4. Rear Foot Pain

Case History - Rear Foot Pain

A 26-year-old 800 m runner developed pain over the posterior calcaneus during the competitive
indoor season. The pain had a gradual onset but now had become severe enough to limit the
frequency and intensity of his training. Symptoms were associated with the commencement of
speed training and to switching from training shoes to spikes (with a reduced heel wedge). He
found that by taking a slower and a longer warm up, he could initially continue to train. He noted
a marked stiffness and a squeaking sensation once he cooled down approximately an hour after
training. He was particularly stiff and painful when he got up each morning.
4 - Injuries to the Musculoskeletal

Discussion
Pain at or near the calcaneus can be a frustrating problem for the athlete and the physician/
physiotherapist. A relatively small problem can produce total disability. Proper diagnosis and treatment
are essential for success.
System

Differential diagnoses include:


a. stress fracture of the calcaneus
b. osteoid osteoma of the calcaneus
c. medial plantar nerve, calcaneal nerve, or tibial nerve entrapment
d. flexor hallucis longus, flexor digitorum longus, or tibialis posterior tendonitis
e. Achilles tendonopathy, retrocalcaneal bursitis, and partial or complete rupture of the Achilles
tendon
f. talar tubercle compression/injury (medial or lateral)
g. enthesitis

Location of the pain is an important feature in reaching a proper diagnosis. When the athlete localizes
the discomfort to the body or plantar surface of the calcaneus, several possibilities exist. In the adolescent
athlete, Severs disease could present in this manner. In the adult athlete with pain on medial to lateral
compression, tenderness to the body of the calcaneus may be caused by stress fracture, infection, or
- 228 -
IOC Sport Medicine Manual 2000
neoplasm. Imaging techniques such as x-ray, radionuclide scan and CT scan may be necessary to
distinguish these problems. A gallium scan is specific for osteomyelitis. A CT scan usually will reveal
the characteristics of an osteoid osteoma. Treatment for these two conditions usually involves surgery
but a stress fracture will usually respond to withdrawal of weight-bearing activity. Medial or lateral
talar tubercle injury can be diagnosed with similar testing procedures but now oriented on the posterior
talus.

Pain on the plantar surface of the heel associated with the paraesthesia, a possible positive Tinels sign
and possible wasting of innervated foot muscles should alert the physician to the possibility of
entrapment of the medial plantar nerve, calcaneal nerve or tibial nerve. Nerve mobility may be assessed
and addressed as indicated. Tape or orthotic support, followed by oral anti-inflammatories and local
corticosteroid injections comprise the usual treatment.

When the pain is anteromedial to the insertion of the Achilles tendon and pain is precipitated by
plantar flexion and 1st metatarsal flexion, flexor hallicus longus tendonitis should be considered.
Resisted plantar flexion of the hallux will amplify pain and a snap may be elicited as well. This
injury is typical in the dancer working en pointe. Differential diagnosis of flexor digitorum longus
muscle (flexion of metatarsal 2-5) and tibialis posterior muscle (ankle inversion and plantar flexion)
can be done. Modified rest, oral anti-inflammatories and local corticosteroid injection in the peritendon
area may be indicated. As soon as possible, gentle progressive stretching, concentric and eccentric
training of the muscles in question, as well as associated agonist and antagonist muscles should be
started. This would be in non-weight-bearing initially, and as pain diminishes, can be progressed to
weight-bearing (see Ankle and Foot 1. Ankle, 2. Medial Foot pain case histories).

The most common cause of rear foot pain, however, involves the Achilles tendon and its bursa. Major
loading to a normal or abnormal tendon may result in Achilles tendon rupture, while repeated minor
trauma to a tendon may result in Achilles tendonopathy.

There are many causes of Achilles tendon pain. Certain cases deserve special attention:
 the presence of a systemic inflammatory disorder (eg. rheumatoid disease, ankylosing
spondylitis or gout)

4 - Injuries to the Musculoskeletal


 iatrogenic lesions, especially following steroid administration. It has been well documented
that injection of steroid into the tendon can lead to an alteration of the tissue structure, and
possibly failure
 previous trauma, such as previous surgery, or scarring around the tendon from laceration or

System
infection

Achilles tendonopathy is a term used to generally describe acute and chronic injuries to the Achilles
tendon in-substance or to its sheath covering, the paratenon. The most common cause of Achilles
tendon injury is repeated minor trauma arising from repeated loading. The relatively avascular
watershed area, which is two to six centimetres proximal to the calcaneal insertion, is particularly
vulnerable. Acute Achilles tendon injury is typically characterized by focal tenderness, swelling, and
a palpable tendon nodule. If healing is inadequate and tendon injury re-occurs, then degenerative
changes accumulate (Achilles tendonosis) and partial tears or complete ruptures may result.
Inflammation of the paratenon (paratenonitis) may also arise from repetitive loading of the Achilles
tendon. Symptoms and signs are similar to that of in-substance tendon injury, except that crepitus
may also be present. With repetitive injury, paratenon thickening occurs. Ultrasound imaging is useful
in distinguishing injuries of the tendon in-substance or paratenon and to visualize degenerative changes
and/or tears.

- 229 -
IOC Sport Medicine Manual 2000
Once contributing systemic inflammatory disorders have been ruled out, initial treatment of Achilles
tendonopathy includes:
 relative rest and cross training to avoid further injury
 gentle stretching of the calf muscles to maintain flexibility
 ice, oral or topical anti-inflammatory medication, and physiotherapy soft tissue techniques
and modalities to reducing swelling and pain

Once acute symptoms have improved, an eccentric heel drop strengthening drill is started (see Ankle
Injuries). If symptoms continue to persist, deep friction massage, taping (Figure 4.70) and/or in-shoe
orthotics may be helpful.

Figure 4.69 Achilles taping.

Chronic cases of tendonopathy that do not respond to non-operative treatment may require surgical
intervention. Peritendonous stripping and in-substance tendon debridement may promote tendon
healing and resolution of pain. Post-operatively, the tendon is protected and activities are gradually
reintroduced over a period of 57 months:
 mobilization with non-weight-bearing plus hydrotherapy and the use of heel lifts
 full weight-bearing and non-impact stress; slow introduction of a stretching and eccentric
strength programme
 return to full activity and loading (see Ankle and Foot 1. Ankle and 2. Medial Foot pain case
4 - Injuries to the Musculoskeletal

histories)

Partial ruptures of the Achilles tendon can be treated using the previously mentioned rehabilitation
programme for the post-operative Achilles tendon.
System

Complete rupture of the Achilles tendon may be treated with immobilization of the ankle in varying
degrees of plantar-flexion over 6-10 weeks, followed by the above rehabilitation programme for the
post-operative Achilles tendon. The outcome of conservative treatment is similar to surgical repair
except that there may be a higher rate of re-rupture, particularly in athletes.

- 230 -
IOC Sport Medicine Manual 2000
Achilles tendinopathy

Control of
Rehabilitation of Control of inflammation
biomechanical
muscle-tendon unit and pain
parameters

1. Stretching 1. Ice massage 1. Corrective in-shoe


2. Strengthening - 2. Modified rest* orthotic
with eccentric 3. NSAID (oral or 2. Taping
exercises topical) 3. Change of shoe
4. Physiotherapy

*Modified Rest:
a. Reduction of training intensity and volume
b. Introduction of non-impact loading activities

Figure 4.70 Treatment strategies for Achilles tendinopathy.

Retrocalcaneal bursitis is distinguished from Achilles tendonopathy by focal tenderness and swelling
anterior to the tendon and just superior to the calcaneal insertion.

4 - Injuries to the Musculoskeletal


Achilles Tendon

System
Calcaneus
Retrocalcaneal Bursitis

Figure 4.71 Retrocalcaneal bursitis may result from external pressure or from compression secondary
to swelling in the Achilles tendon or peritendon.

Treatment of retro-calcaneal bursitis is directed at protecting the area from pressure with a doughnut
and punching-out the boot or shoe.

The 800 m runner was found to have acute Achilles peritenonitis. Following the conservative
management outlined above, he resumed training in six weeks time with no complaints of pain.

- 231 -
IOC Sport Medicine Manual 2000
I. References
For futher information, please refer to the following web sites:
http://ESPN.go.com/trainingroom/s/injury/archive.html
http://www.hockeyinjuries.com
http://www.vh.org/Providers/Textbooks/MuscleInjuries/MuscleInjuries.html
http://www.KneeOne.com
4 - Injuries to the Musculoskeletal
System

- 232 -
IOC Sport Medicine Manual 2000

Potrebbero piacerti anche